Anda di halaman 1dari 28

Pediatrics 01May2009DO NOT DISTRIBUTE - 1 -Pediatrics #1 Clinical: Part One1) Wh at is the most common etiology of pediatric cardiac arrest?

?a) Respiratory b) Car diacc) CNSd) Metabolice) Renal2) When assessing a pediatric patient in cardiores piratory arrest, the primary surveyinvolves ABCDE (airway, breathing, circulatio n, disability, and exposure). Which of thefollowing components of the primary su rvey is contraindicated in children?a) Head-tilt chin-lift b) Jaw-thrust maneuve r c) Blind finger sweepd) Using heart rate to assess intravascular volume status e) Using capillary refill to assess adequate circulation3) Which of the followin g is a primary treatment option for a pediatric patient in pulseless ventricular tachycardia (VT) or ventricular fibrillation (VF)?a) Cardioversion b) Amiodaron e or procainamidec) Vagal maneuvers and adenosined) Atropine and epinephrinee) D efibrillation4) A child presents with a cardiac arrhythmia. The Emergency Depart ment physician putsa bag of ice-cold water on the childs face to induce the mamma lian diving reflex (vagalmaneuver). Which of the following did the patient li el y have?a) Hemodynamically stable VT b) Hemodynamically stable SVTc) Hemodynamica lly unstable VT or SVTd) Pulseless VT or VFe) Bradycardia or asystole5) Which of the following drugs would be useful for multiple ventricular ectopy (PVCs)as we ll as for refractory VT/VF to improve the susceptibility for cardioversion?a) At ropine b) Bicarbonatec) Calcium and magnesiumd) Lidocainee) Epinephrine6) What i s the most common type of shoc seen in children?a) Hypovolemic (electrolyte los s, blood loss, third spacing) b) Cardiogenic (congenital heart disease, cardiomy opathy)c) Disruptive (anaphylactic, neurologic, drug toxicity)d) Septic, compens ated (warm)e) Septic, uncompensated (cold)7) Approximately what percentage of childh ood poisonings occur in children younger than 5 years of age?a) 15% Pediatrics 01May2009DO NOT DISTRIBUTE - 2 - b) 35%c) 50%d) 65%e) 85%8) A child p resents with irritability, anorexia, vomiting, and hyperactivity. The clinicians uspects lead poisoning and draws labs loo ing for >20mcg/dL, but the labs return >45mcg/dL so the clinician immediately begins chelation treatment. What is the m ostli ely source of this childs poisoning?a) Playground equipment b) Paint from a n old buildingc) Mens multivitamin tabletsd) Window blindse) Graphite pencils9) W hat is the leading cause of accidental death in children older than 1 year andad olescents?a) Drowning b) Lead poisoningc) Burnsd) Motor vehicle accidentse) Fore ign body aspiration10) Which of the following pediatric patients is most li ely to be involved in a drowningaccident (bathtub submersion)?a) Caucasian males b) Caucasian femalesc) African American malesd) African American females11) Foreign body aspiration has the highest incidence in children aged 6-30 months.What is the most common object involved, accounting for over 50% of aspirations?a) Small toys (e.g. lego, marble) b) Nutsc) Breadd) Crayonse) Fish bones12) Burns are th e 3rd leading cause of injury in children and the 2nd most frequent causeof acci dental death. They involve abuse in 15-25% of cases. What is the most commontype of burn seen in children?a) Scald burns b) Flame burnsc) Electrical burnsd) Con tact burnse) Chemical burns13) Which of the following is NOT true of child abuse ?a) Neglect is the most common cause of failure to thrive b) Sexual abusers of c hildren are usually stepfathers, fathers, or other male familymembersc) 10% of E R visits involving children younger than 5 years old result from abuse Pediatrics 01May2009DO NOT DISTRIBUTE - 3 -d) 80% of sexual abuse victims are gi rlse) Molestation by strangers and babysitters is common14) It has been shown th at home apnea monitors do not decrease the li elihood of suddeninfant death synd rome (SIDS). Which of the following is NOT a ris factor for SIDS?a) Prematurity b) Low birth weightc) Female gender d) Prone sleeping positione) Intrauterine g rowth restriction (IUGR)f) Maternal smo ing during pregnancy15) Newborn cyanosis is most li ely due to cardiac, pulmonary, neurologic, or hematologic disorders. It is clinically evident when absolute concentrations of deoxygenated hemoglobi n rise above:a) 1g/dL b) 2g/dLc) 3g/dLd) 4g/dLe) 5g/dL16) Which of the following should be given to an unstable infant with suspectedcongenital heart disease?a) Prostaglandin E1 (alprostadil) b) Prostaglandin E2 (dinoprostone)c) Phosphodies terase 3 inhibitor (milrinone)d) Phosphodiesterase 4 inhibitor (mesembrine)e) Ph osphodiesterase 5 inhibitor (sildenafil)17) During a routine cardiac examination of an infant, a murmur is heard. Further examination reveals a fixed split S2,

regardless of inspiration or expiration. An ECGshows mild right ventricular hype rtrophy (RVH). Which of the following is most li ely?a) Atrial septal defect b) Ventricular septal defectc) Patent ductusd) Tetrology of Fallote) Coarctation of the aortaf) Transposition of the great arteries18) Which of the following prese nts with a holosystolic murmur and is the most commoncongenital heart defect?a) Patent ductus b) Atrial septal defectc) Tetrology of Fallotd) Coarctation of the aortae) Ventricular septal defectf) Transposition of the great arteries19) A co ntinuous machinery murmur is heart in which of the following?a) Patent ductus b) A trial septal defectc) Tetrology of Fallot Pediatrics 01May2009DO NOT DISTRIBUTE - 4 -d) Coarctation of the aortae) Ventric ular septal defectf) Transposition of the great arteries20) A young child presen ts with cyanosis. The mother says the child has periodicepisodic cyanosis, which is accompanied by agitation and rapid, deep breathing. Thechild turns blue for a couple minutes then normal color returns. Chest x-ray shows aboot shaped heart a nd ECG shows right-axis deviation. Which of the following would NOT be seen in t his child?a) Over-riding aorta b) Atrial septal defectc) Right ventricular hyper trophyd) Pulmonary stenosis21) A newborn is found to have Turner syndrome. Blood pressure measurements show adifference between upper and lower extremities. Whi ch of the following is most li ely?a) Patent ductus b) Atrial septal defectc) Te trology of Fallotd) Coarctation of the aortae) Ventricular septal defectf) Trans position of the great arteries22) A newborn boy presents to the Emergency Room w ith cyanosis. A chest x-ray showsan egg-on-a-string (egg-shaped) heart. Which of the following is most li ely?a) Patent ductus b) Atrial septal defectc) Tetrolo gy of Fallotd) Coarctation of the aortae) Ventricular septal defectf) Transposit ion of the great arteries23) Conotruncal anomalies (e.g. truncus arteriosus, tet ralogy of Fallot, VSD) areassociated with what chromosomal micro-deletion?a) 11p 22 b) 11q22c) 22p11d) 22q1124) What is the most common cardiac lesion found in r heumatic heart disease?a) Hypoplastic left heart syndrome b) Coronary artery ane urysmsc) Aortic stenosisd) Mitral regurgitatione) Ventricular septal defect25) A young child is found to have pericarditis, myocarditis, and transient rhythmdis turbances. If Kawasa i disease is suspected, which of the following may develop? a) Hypoplastic left heart syndrome b) Coronary artery aneurysmsc) Aortic stenosi sd) Mitral regurgitation Pediatrics 01May2009DO NOT DISTRIBUTE - 5 -e) Ventricular septal defect26) Endoc arditis is seen in adult IV drug abusers and with prosthetic replacement valvesa fter dental procedures. In children, it is seen with congenitally abnormal valve s. What isthe most common causative agent?a)Staphylococcus aureus b)Streptococc us viridans c) Haemophilus influenzae(HACEK)d)Staphylococcus epidermidis e) Coxs ac ie B virus27) A child presents with fever, dyspnea, tachycardia, and mild STand T-wave changeson ECG. Physical exam reveals S3 ventricular gallop. If myocar ditis is suspected, whichof the following is most li ely?a)Staphylococcus aureus b)Streptococcus viridans c) Haemophilus influenzae(HACEK)d)Staphylococcus epid ermidis e) Coxsac ie B virus28) Dyspnea, orthopnea, paroxysmal nocturnal dyspnea (PND), pulsus alternans, andneuromuscular disease etiology (e.g. Duchenne Muscu lar Dystrophy) is seen in which of the following?a) Hypertrophic cardiomyopathy b) Dilated cardiomyopathyc) Endocarditisd) Myocarditise) Myocardial infarction29 ) A young high school bas etball player suddenly collapses during a game. Which of the following is most li ely?a) Hypertrophic cardiomyopathy b) Dilated cardio myopathyc) Endocarditisd) Myocarditise) Myocardial infarction30) Which of the fo llowing developmental tests evaluates children from 0 to 6 years of age and invo lves gross motor, fine motor-adaptive, language, and personal-social?a) Denver I I b) CATc) CLAMSd) B & C31) What is the best indicator of future intellectual ac hievement?a) Gross motor b) Fine motor-adaptivec) Languaged) Personal-social32) A development quotient below what level would constitute developmental delay?a) 100 b) 90 Pediatrics 01May2009DO NOT DISTRIBUTE - 6 -c) 80d) 70e) 6033) Which of the follo wing intelligence quotients (IQs) would signify severely mentallyretarded?a) 80 b) 60c) 45d) 30e) 1034) Which of the following is NOT true of attention deficit hyperactivity disorder (ADHD)?a) Most of those affected as a child do not have p ersisting symptoms b) Involves hyperactivity, inattention, and impulsivityc) Req

uires information from parents and teachers using a scale such as theConner Pare nt and Teacher Scalesd) Symptoms must be present for at least 6 monthse) Treatme nt may include drugs such as methylphenidate (Ritalin),dextroamphetamine (Dexedr ine) and mixed amphetamine salts (Adderall)35) A child is able to balance on one foot, can copy a cross, can catch a ball, can dresshimself, and tells an intell igible story to a stranger. How old is this child?a) 1-year b) 2-yearsc) 3-year sd) 4-yearse) 5-years36) A child is able to discriminate the use of mama and dada, s tands alone,cooperates with dressing, and imitates actions. How old is this chil d?a) 36-months b) 24-monthsc) 12-monthsd) 6-monthse) 2-months37) A child can rol l in both directions, sits still, reaches with one hand, and babbles. Howold is this child?a) 2-months b) 4-monthsc) 6-monthsd) 9-monthse) 12-months38) Autism a nd Asperger syndrome are pervasive development disorders (PDD).Asperger is chara cterized by intense interest in specific topics (e.g. dinosaurs) as well aswhich of the following?a) Lac of language ability b) Lac of showing interestc) Lac of emotional reciprocity Pediatrics 01May2009DO NOT DISTRIBUTE - 7 -d) Lac of abstract form understandin g (e.g. sarcasm)e) Lac of motor mannerisms and inflexibility to rituals39) The measles-mumps-rubella (MMR) vaccine and thimerosal have been shown to beassociat ed with the development of autism.a) True b) FalseMatch the disease with the cau sative agent: 40) Measles a) Parvovirus B1941) Rubella b) Paramyxovirus42) Roseo la infantum c) Varicella-Zoster virus43) Fifth disease (erythema infectiosum) d) RNA Togavirus44) Hand-Foot-and-Mouth e) HHV-645) Chic en pox f) Coxsac ie A vir usMatch the disease with the type of rash:46) Measles a) Mouth ulcers, football shaped vesicles47) Rubella b) Pruritic rash, red papules, vesicles, scabs48) Ros eola infantum c) Maculopapular on trun to periphery49) Fifth disease (erythema infectiosum) d) Discrete maculopapular, <5 days50) Hand-Foot-and-Mouth e) Kopli spots, maculopapular at head51) Chic en pox f) Slapped-chee , arms spread to tr un 52) An infant presents with abrupt onset of diffuse erythema, mar ed s in te nderness,and fever. Flaccid bullae develop and rupture, leading beefy red, weepi ng surfaces.Physical exam reveals separation of the epidermis on light rubbing ( Ni ols y sign). LabsshowS. aureusas the causative agent. Which of the following is most li ely?a) Bullous impetigo b) Nonbullous impetigoc) Scalded s in syndrom ed) Folliculitise) Cellulitis53) A child presents with a hyperpigmented scaly le sion on the upper bac , chest, andnec . Labs are negative for septated branching hyphae. However, a spaghetti andmeatballs arrangement is seen on microscopy. Furt her lab testing showsMalassezia furfur . Which of the following is most li ely?a )Tinea capitis b)Tinea corporis c)Tinea cruris d)Tinea pedis e)Tinea versicolor 54) A boy with oily s in presents with complaints of facial blemishes that bega n as hereached puberty. The clinician believes Propionibacterium acnesis to blam e. Whattreatment option is reserved for severe cases that involve inflammatory p apules, cysts,abscesses, and scaring?a) Benzoyl peroxide b) Topical antibiotics (clindamycin)c) Topical retinoids (tretinoin)d) Oral antibiotics (tetracycline) Pediatrics 01May2009DO NOT DISTRIBUTE - 8 -e) Oral retinoic acid (Accutane)55) A child is found to have a silvery scale rash on areas of physical or thermal tra uma(Kbner phenomenon). The child scratches at the rash, which causes pinpoint ble eding(Auspitz sign). Which of the following is most li ely?a) Acne b) Tinea corp orisc) Folliculitisd) Psoriasise) Bullous impetigo56) Which of the following is NOT characteristic of diabetic etoacidosis?a) Type II diabetes b) Polyuria, pol ydipsiac) Kussmaul breathingd) Tachycardia and hypotensione) Fruity breath and c erebral edema57) An infant girl is found to have short statue and deformities of the nec , chest, andheart. Physical exam reveals a nuc le- nuc le-dimple-dimpl e sign. Which of thefollowing is most li ely?a) Central diabetes insipidus b) Gr owth hormone deficiencyc) Primary hypothyroidismd) Cushing syndromee) Turner syn drome58) What is the most common cause of hyperthyroidism in children?a) Hashimo to thyroiditis b) Graves diseasec) Toxic multinodular goiter d) Amiodarone toxic itye) Knuc le- nuc le-dimple- nuc le pseudohypothyroidism59) What is the most co mmon cause of congenital adrenal hyperplasia?a) 21-hydroxylase deficiency b) 17a lpha-hydroxylase deficiencyc) Aromatase deficiencyd) 11beta-hydroxylase deficien cye) 3beta-hydroxylase deficiency60) Precocious puberty is defined as developmen

t of secondary sex characteristics beforeage 7.5 in girls and before what age in boys?a) 5 b) 6c) 7.5d) 9e) 1061) A child presents with wea ness, nausea, vomiti ng, and weight loss. History revealssalt cravings. Physical exam reveals increas ed s in pigmentation and posturalhypotension. Lab testing shows hyponatremia, hy per alemia, hypoglycemia, andmetabolic acidosis. Which of the following is most li ely? Pediatrics 01May2009DO NOT DISTRIBUTE - 9 -a) Cushing syndrome b) Cushing diseas ec) Addison diseased) Pubertal delaye) Pseudopseudohypothyroidism62) What is the Holliday-Segar method used for?a) Fluid bolus calculation in burn victims b) Fl uid infusion rate for traumatic injuriesc) Fluid amount for daily maintenanced) Fluid allowance for diabetes insipiduse) Fluid loss rate for dehydration63) A ch ild presents with suspected dehydration due to immature idneys. Evaluationshows a depressed anterior fontanelle, sun en eyes, altered mental status, and increa sedheart rate. It is estimated that the child has lost 7% of their body weight t o water. Labsreveal 800mOsm/L urine, slightly elevated urine specific gravity, e levated BUN, andmild acidosis. Which of the following best describes this patien t?a) Normal b) Mild shoc c) Compensated shoc d) Uncompensated shoc 64) The he molysis of red blood cells during sample collections is the most commoncauses of (artifactual):a) Hyponatremia b) Hypernatremiac) Hypo alemiad) Hyper alemia65) A child presents with constipation after a bout of severe vomiting. The clinicia n noteswea ness and tetany. An ECG shows depressed ST segments with biphasic T-w aves and prominent U-waves. Which of the following is most li ely?a) Hyponatremi a b) Hypernatremiac) Hypo alemiad) Hyper alemia66) A child with a congenital ren al tubule defect has large losses of bicarbonate. Labsreveal a blood pH of 7.20. Which of the following is most li ely?a) Metabolic acidosis b) Metabolic al alo sisc) Respiratory acidosisd) Respiratory al alosisPediatrics #2 Clinical: Part T wo1) What is the most common medical cause of abdominal pain in children?a) Sic le cell disease b) Lactase deficiencyc) Gastroenteritisd) Mesenteric lymphadenit is Pediatrics 01May2009DO NOT DISTRIBUTE - 10 -e) Group A strep infections2) What i s the most common surgical cause of abdominal pain in children?a) Trauma b) Intu ssusceptionc) Cholecystitisd) Appendicitise) Testicular torsion3) Which of the f ollowing cases would suggest colic y pain or obstruction?a) Blood or mucinous di arrhea b) Child lies stillc) Child cannot remain stilld) Sore throate) Dysuria4) Which of the following cases would suggest inflammatory pain, infection, or pe rforated organ/viscus?a) Blood or mucinous diarrhea b) Child lies stillc) Child cannot remain stilld) Sore throate) Dysuria5) An infant presents with violent ep isodes of irritability, colic y pain, and emesis thatare interspersed with relat ively normal periods. A barium enema is performed, showing acoiled-spring appear ance to the bowel. Which of the following is most li ely?a) Trauma b) Intussusce ptionc) Cholecystitisd) Appendicitise) Testicular torsion6) Which of the followi ng is an uncommon cause of emesis in children?a) Crohn disease b) Gastroesophage al refluxc) Acute gastroenteritisd) Tonsillitise) Otitis media7) A three-wee -ol d child presents with non-bilious projectile vomiting. Which of thefollowing is most li ely?a) Gastroesophageal reflux b) GI malrotationc) Pyloric stenosisd) Hi rschsprung diseasee) Esophageal stricture8) A newborn is brought in with gastroi ntestinal problems. An upper GI series is performed, showing abnormal positionin g of the ligament of Treitz and the cecum.Which of the following is most li ely? a) Gastroesophageal reflux b) GI malrotation Pediatrics 01May2009DO NOT DISTRIBUTE - 11 -c) Pyloric stenosisd) Hirschsprung d iseasee) Esophageal stricture9) What is the most common cause of acute diarrhea in children within most developedcountries?a) Bacterial enterocolitis b) Appendi citisc) Iron, mercury, lead, or fluoride ingestiond) Intussusceptione) Viral gas troenteritis10) Hirschsprung disease (i.e. congenital megacolon) results from fa ilure of ganglioncells of the myenteric plexus to migrate down to the distal col on. This results in:a) Constipation b) Diarrheac) Hematocheziad) Obstipation11) Which of the following is defined as bright red blood per rectum (BRBPR) andusua lly results from a lower GI bleed, distal to the ligament of Treitz?a) Constipat ion b) Diarrheac) Hematocheziad) Obstipatione) Melena12) Which of the following is the most common anomaly of the GI tract, is due tovestigial remnant of the om

phalomesenteric duct (within 100cm of the ileocecal valve),and presents with pai nless rectal bleeding?a) Gastroschisis b) Mec el diverticulumc) Hirschsprung dis eased) Omphalocelee) Diaphragmatic hernia13) Which of the following is NOT true of congenital defects?a) Autosomal dominant disorders typically code for structu ral proteins b) Autosomal recessive disorders typically code for enzymesc) Envir onmental factors cause 10% of birth defectsd) Major anomalies have a low inciden ce up to newborns with the presence of three minor anomaliese) Ornithine transca rbamylase deficiency (OTC), li e all other inborn errors of metabolism, is autos omal recessiveMatch the genetic defect with the characteristics:14) Horseshoe i dney, roc er-bottom feet, low-set ears a) Downs (21)15) Gynecomastia, small phal lus, small testes, taller b) Edwards (18)16) Wide-spaced hypoplastic nipples, we bbed nec , shield chest c) Patau (13)17) High leu emia ris , short, broad hands, dysmorphic face d) Turner 18) Microcephaly, sloped forehead, cryptorchidism, po lydactyly e) Klinefelter Pediatrics 01May2009DO NOT DISTRIBUTE - 12 -19) A mother brings in her young boy because he is gaining weight rapidly. She has beentrying to eep food away from him, but finds him digging through the garbage and eatingnon-edible items (pica ). Physical exam reveals almond-shaped eyes that are differentfrom the mothers, a down-turned mouth, and small hands and feet. Which of thefollowing is most li e ly?a) Maternal chromosome 15 deletion b) Paternal chromosome 15 deletionc) Triso my 21d) Trisomy 18e) Trisomy 1320) A girl is brought in by her mother with the c oncern that the child is not learning properly. Physical exam reveals maxillary hypoplasia, a large mouth, prognathism, andshort stature. Mental assessment reve als the child is severely mentally retarded. During adiscussion with the mother, the clinician notes the child ma ing jer y arm movements,marionette-li e moveme nts, and laughing a great deal (happy puppet syndrome). Whichof the following is most li ely?a) Maternal chromosome 15 deletion b) Paternal chromosome 15 deleti onc) Deletion of 22q11d) Deletion of 22p11e) Trisomy 2121) A newborn is found to have macrosomia at birth. Testing as the child begins to growreveals macroorchi dism, large ears, and a large jaw. Genetic testing reveals a largeamount of CGG trinucleotide repeats. Which of the following is most li ely?a) Angelman syndrom e b) Prader-Willi syndromec) Fragile X syndromed) DiGeorge syndromee) Hurler syn drome22) Microdeletion of 22q11.2 can lead to tetralogy of Fallot, interrupted a ortic arch, andthymus absence. It is associated with velocardiofacial syndrome a s well as:a) Angelman syndrome b) Prader-Willi syndromec) Fragile X syndromed) D iGeorge syndromee) Hurler syndrome23) A 2-wee -old presents with vision problems , anorexia, and emesis after breast-feeding. Testing reveals liver failure, rena l dysfunction, and developing cataracts. Adeficiency in G1P uridyltransferase is found. The clinician is concerned about the ris of E. colisepsis. Which of th e following is most li ely?a) OTC deficiency b) Gaucher diseasec) Hurler syndrom ed) Phenyl etonuriae) Galactosemiaf) Homocystinuria Pediatrics 01May2009DO NOT DISTRIBUTE - 13 -Match the glycogen storage disease w ith the deficiency: 24) Acid alpha-glucosidase (GAA) a) Von Gier e disease (Type I)25) Glucose-6-phosphate (G6P) b) Pompe disease (Type II)26) Myophosphorylase c) McArdle disease (Type V)27) A fair-haired, fair-s inned, blue-eyed child pres ents with projectile vomiting, mentalretardation, tremors, and mouse-li e smelli ng urine. Testing reveals a defect in tyrosineformation. Which of the following is most li ely?a) OTC deficiency b) Gaucher diseasec) Hurler syndromed) Phenyl e tonuriae) Galactosemiaf) Homocystinuria28) Which of the following can cause eye lens dislocation and involves a defect in the pathway that converts methionine t o cysteine and serine?a) OTC deficiency b) Gaucher diseasec) Hurler syndromed) P henyl etonuriae) Galactosemiaf) Homocystinuria29) A patient is diagnosed with OT C deficiency after developing severehyperammonemia after the consumption of prot eins. How is this disorder inherited?a) Autosomal dominant b) Autosomal recessiv ec) X-lin edd) Y-lin ede) Mitochondrial30) Which of the following is due to a de ficiency in alpha-iduroindase leading todermatan and heparan sulfate accumulatio n, coarse facies, corneal clouding, andexaggerated yphosis?a) OTC deficiency b) Gaucher diseasec) Hurler syndromed) Phenyl etonuriae) Galactosemiaf) Homocystin uria31) Which of the following is due to a deficiency in beta-glucosidase leadin g toglucocerebrosidase accumulation, anemia, leucopenia, and an Erlenmeyer flas

-shape of the distal femur?a) OTC deficiency b) Gaucher diseasec) Hurler syndrom ed) Phenyl etonuriae) Galactosemiaf) Homocystinuria Pediatrics 01May2009DO NOT DISTRIBUTE - 14 -32) An infant presents with anorexia , apathy, easy fatigability, and irritability. Physicalexam reveals s in pallor, glossitis, and oilonychias. Labs reveal iron deficiency. Whichof the following is most li ely?a) Hypochromic microcytic anemia b) Hyperchromic microcytic anem iac) Hypochromic macrocytic anemiad) Hyperchromic macrocytic anemiae) Normocytic anemia33) During the first year of life, a child develops severe hemolytic anem ia andsplenomegaly. A physical exam reveals a tower s ull, frontal bossing, and prominentchee bones. Labs are negative for excess beta-globin tetramers. The chi ld is started onfolate supplementation, RBC transfusions, and iron chelation. Wh ich of the following ismost li ely?a) Homozygous alpha thalassemia (Bart) b) Hem oglobin H diseasec) Alpha thalassemia minor d) Beta thalassemia major e) Beta th alassemia minor 34) A 2-year-old develops bone marrow suppression after a viral infection. Pure red cellaplasia is found and the child is diagnosed with transie nt erythroblastopenia of childhood(TEC). The mother is told it will pass. What t ype of anemia is this?a) Hypochromic microcytic anemia b) Hyperchromic microcyti c anemiac) Hypochromic macrocytic anemiad) Hyperchromic macrocytic anemiae) Norm ocytic anemia35) An African-American child presents with pain in the hands, nee s, and a generalaching sensation. Physical exam reveals splenomegaly, dactylitis , and pulmonary rales.Labs show anemia. The father has a history of some type of RBC disorder. Which of thefollowing does the child most li ely have?a) Beta-glo bin position 4 substitution of glutamine for valine b) Beta-globin position 4 su bstitution of valine for glutaminec) Beta-globin position 6 substitution of glut amine for valined) Beta-globin position 6 substitution of valine for glutamine36 ) A blood test on a child reveals Heinz bodies. Further testing reveals G6PD, wh ich is alac of the hexose monophosphate shunt pathway enzyme that results in a depletion of NADPH and inability to regenerate reduced glutathione. How is this disorder inherited?a) Autosomal dominant b) Autosomal recessivec) X-lin edd) Ylin ede) Mitochondrial37) A child is brought to the emergency center with trauma tic hemorrhages after a soccer game. Physical exam reveals significant nee hema rthroses. Further testing reveals an X-lin ed recessive disorder involving facto r VIII deficiency. Which of the following is mostli ely? Pediatrics 01May2009DO NOT DISTRIBUTE - 15 -a) Sic le cell disease b) Alpha thal assemiac) Hemophilia Ad) Hemophilia Be) Von Willebrand disease38) A child presen ts with complaints of bleeding gums when she brushes her teeth andeven when she eats. Testing reveals a deficiency in von Willebrand factor. What is thedrug of choice for this patient for bleeding episodes?a) Vitamin K b) Clopidogrelc) Des mopressin acetated) Cryoprecipitatee) Fresh-frozen plasma39) What clotting facto r is deficient in hemophilia B?a) Factor V b) Factor VIIc) Factor VIIId) Factor IXe) Factor X40) Which of the following is NOT true?a) Hypocalcemic tetany with absence of the thymic shadow and cell-mediatedimmunodeficiency suggests 22q11 de letion b) Humoral immunodeficiency predisposes patients to infections withencaps ulated organisms (e.g. H. influenzae ,S. pneumonia)c) Cell-mediated immunodeficiency predisposes patients to autoimmu nedisorders, intracellular organisms, and opportunistic infectionsd) Typical sig ns of infections are often absent in the presence of neutropeniae) The most comm on immunodeficiency seen in pediatrics is cell-mediated41) A patient presents wi th recurrent infections involving catalase-producing bacteria (S.aureus ,C. albicans, Aspergillus). A nitroblue tetrazolium test (NBT) anddihydrorhodami ne reduction (DHR) test are positive for the detection of chronicgranulomatous d isease (CGD). Along with interferon-gamma, what prophylactic dailydrug should th is patient receive?a) Penicillin G b) Amphotericin Bc) Doxycyclined) TMP-SMXe) I soniazid42) A child presents with seasonal rhinorrhea and upper respiratory trac t symptoms. Hehas dar circles under his eyes and a horizontal crease across the middle of the nose. AType I hypersensitivity response is suspected. What is the treatment of choice?a) Sedating H1 bloc ers (e.g. diphenhydramine) b) Non-sedat ing H1 bloc ers (e.g. fexofenadine)c) H2 bloc er (e.g. famotidine)d) Corticoster oid (e.g. prednisone)e) Antibiotic (e.g. amoxicillin)

Pediatrics 01May2009DO NOT DISTRIBUTE - 16 -43) What is the gold standard for di agnosis of food allergies, such as to peanuts, eggs,mil , soy, wheat, or fish?a) Patient history b) Allergy scratch testc) Allergy blood paneld) H1 suppression teste) Double-blind placebo-controlled food challenge44) A 13-year-old girl pres ents with joint pain lasting for 2-months. She admits to jointstiffness in the m orning that resolves with movement. Her mother says she has beeneating less, fat igues easier, and is irritable. Physical exam reveals a rash and limited rangeof motion in the affected joints. Which of the following is most li ely?a) Systemi c lupus erythematosus (SLE) b) Polyarteritis nodosa (PAN)c) Kawasa i diseased) J uvenile rheumatoid arthritis (JRA)e) DiGeorge syndrome45) A late adolescent pres ents with a malar facial rash, photosensitivity, and painless oralulcerations. T esting reveals anemia, positive ANA, and signs of a type IIIhypersensitivity rea ction. What is the treatment of choice?a) Sedating H1 bloc ers (e.g. diphenhydra mine) b) Non-sedating H1 bloc ers (e.g. fexofenadine)c) H2 bloc er (e.g. famotid ine)d) Corticosteroid (e.g. prednisone)e) Antibiotic (e.g. amoxicillin)46) PAN u sually presents with waxing and waning symptoms of painful erythematouss in nodu les, purpura, hypertension, hematuria, and systemic complaints. Henoch-Schnlein p urpura is an IgA-mediated vasculitis that pea s in winter months, is usually pre ceded by a group A streptococcal upper respiratory infection, and causes non-thr ombocytopenic palpable purura. What is the treatment of choice for both of these ?a) Sedating H1 bloc ers (e.g. diphenhydramine) b) Non-sedating H1 bloc ers (e.g . fexofenadine)c) H2 bloc er (e.g. famotidine)d) Corticosteroid (e.g. prednisone )e) Antibiotic (e.g. amoxicillin)47) An infant presents with high fever, lymphad enopathy, and mucocutaneous lesions.Physical exam reveals bilateral conjunctivit is, dry fissured lips, strawberry tongue,indurative edema of the feet, and trunc al polymorphous rash. Kawasa i disease issuspected. What is the treatment of cho ice for the convalescent phase?a) Acetaminophen b) Aspirinc) Indomethacind) Ibup rofene) Hydrocodone48) Fever of un nown origin (FUO) implies a fever greater tha n 38.3C degrees (101F)for equal to or greater than how many days?a) 3 days Pediatrics 01May2009DO NOT DISTRIBUTE - 17 - b) 5 daysc) 7 daysd) 14 dayse) 21 d ays49) The most common bacteria implicated in acute otitis media (AOM) in pediat rics areS. pneumoniae, H. influenzae, andM. catarrhalis. What is the treatment o f choice?a) Amoxicillin b) Doxycyclinec) Penicillind) Ketoconazolee) Ceftriaxone 50) In general, children with pharyngitis should not be treated with antibiotics empirically as most episodes are viral. If Group A Streptococcus (GAS) is suspec ted,what is the drug of choice?a) Amoxicillin b) Doxycyclinec) Penicillind) Keto conazolee) Ceftriaxone51) A 14-year-old boy presents with extreme fatigue. Testi ng shows fever, generalizedlymphadenopathy, atypical lymphocytes, and a positive heterophile antibody test.Epstein-Barr virus is suspected. Which of the followi ng is most li ely?a) Croup b) Epiglottitisc) Mononucleosisd) Bronchiolitise) Per tussis52) A child arrives at the Emergency Department with a hoarse voice and ba r y seal-li ecough. Physical exam reveals stridor and the clinician orders stero ids and nebulizedepinephrine. Parainfluenza virus is suspected. Which of the fol lowing is most li ely?a) Croup b) Epiglottitisc) Mononucleosisd) Bronchiolitise) Pertussis53) What is the treatment of choice for epiglottitis?a) Supportive onl y b) IV epinephrinec) IV prednisoned) IV amoxicilline) Endotracheal intubation54 ) Bronchiolitis is characterized by 5-10 days of wheezing, rhonchi, and crac les typically between November and April. Prophylactic use of palivizumab may be ben eficial. What is the most common cause of bronchiolitis in children?a) Influenza virus b) Parainfluenza virus Pediatrics 01May2009DO NOT DISTRIBUTE - 18 -c) Respiratory syncytial virusd) Eps tein-Barr viruse)S. pneumoniae55) A child presents to the Emergency Department w ith a long, stridorous inspirationafter a paroxysmal whooping cough. The child is started on erythromycin estolate, amacrolide. Which of the following is most li ely?a) Croup b) Epiglottitisc) Mononucleosisd) Bronchiolitise) Pertussis56) A pa tient is as ed to come into her obstetrician at 35-wee s of gestation to get a b acteria test. The test will help determine if she will receive penicillin during labor to prevent neonatal pneumonia and meningitis. Which of the following is b eing tested for?a)Streptococcus pneumoniae b) Haemophilus influenzaec)Mycoplasma pneumoniaed) Group B strep (S. agalactiae)e) Group D strep ( Enterococcus)57) A

n adolescent child (school age) presents with photophobia, fever, and nec pain. Testing reveals a positive Brudzins i sign and a positive Kernig test. Which of thefollowing is most li ely?a)Streptococcus pneumoniae b) Haemophilus influenzae c)Mycoplasma pneumoniaed) Group B strep (S. agalactiae)e) Group D strep ( Entero coccus)58) Bacterial meningitis involvingS. pneumoniaeor H. influenzaeis most c ommonlyseen in what population?a) Neonates b) Children ages 3-6c) Children ages 6-14d) Teenagerse) Adults59) What is the most common cause of gastroenteritis in infants and toddlers?a)Giardia lamblia b)Campylobacter jejuni c)Yersinia enter ocolitica d) Norwal viruse) RotavirusMatch the form of gastroenteritis with the treatment:60) Shigellosis a) No antibiotics unless systemic61)C. jejuni b) Stop antibiotics, metronidazole62) Salmonella c) TMP/SMX63)C. difficiled) Erythromyc in64) Giardiasis e) Metronidazole Pediatrics 01May2009DO NOT DISTRIBUTE - 19 -65) A child from the South Atlantic U.S. presents with a headache and rash that beganabout a wee after a hi e in th e woods. The rash began on the wrists and an les andspread proximally to the tru n . What is the drug of choice?a) Amoxicillin b) Ceftriaxonec) Penicillind) Doxy cyclinee) Clindamycin66) A five-year-old child is brought in with multiple eryth ema migrans lesions. Physicalexam reveals lymphadenopathy and cranial nerve pals y. ECG shows a slight AV bloc .History reveals the childs brothers brought him in to the Minnesota woods to go hunting, but the child was never examined for tic s after the trip. What is the initial drug of choicefor this child, who is younge r than 8-years-old?a) Amoxicillin b) TMP/SMXc) Ketoconazoled) Doxycyclinee) Clin damycinPediatrics #3 Clinical: Part Three1) What is the Apgar (APGAR) score for a child with a heart rate of 90, irregular andwea cry, cyanotic extremities, we a and slightly flexed extremities, and grimacing facialexpression?a) 0 b) 3c) 5 d) 7e) 92) Which of the following forms of birth trauma involves a diffuse, edem atous, and oftendar swelling of the soft tissues of the scalp that extends acro ss the midline and/or suturelines?a) Erb palsy b) Fractured claviclec) Klump e p alsyd) Caput succedaneume) Cephalhematoma3) A newborn presents with an arm that is extended, internally rotated, and flexed at thewrist. Moro reflex of the righ t arm is absent but right hand grasp is intact. Which of thefollowing is most li ely?a) Erb palsy b) Fractured claviclec) Klump e palsyd) Caput succedaneume) Ce phalhematoma4) Very low birth weight (VLBW) is defined as less than:a) 1,500g Pediatrics 01May2009DO NOT DISTRIBUTE - 20 - b) 2,500gc) 3,500gd) 4,500ge) 5,500 g5) Which of the following is NOT common in postmature (>42 wee s) infants?a) Me conium aspiration b) Persistent pulmonary hypertensionc) Hyperglycemiad) Hypocal cemiae) Polycythemia6) Which of the following congenital infections presents wit h cataracts, blueberrymuffin s in syndrome, vertical bone striation, and patent du ctus?a) Toxoplasma gondii b) Treponema pallidumc) Rubellad) Cytomegaloviruse) He rpes simplex7) Which of the following congenital infections is the most common a nd presents withs in vesicles or denuded s in, eratoconjunctivitis, and seizure s?a) Toxoplasma gondii b) Treponema pallidumc) Rubellad) Cytomegaloviruse) Herpe s simplex8) Early-onset neonatal sepsis usually involves bacteria that colonize the mothersgenitourinary tract. On the other hand, nosocomial-acquired sepsis inv olves drug-resistant bacterial pathogens that are more commonly seen in the neon atal intensive careunit (NICU), such as:a) Group B strep (agalactiae) b) E. coli c) Klebsiellad)S. aureuse) L. monocytogenes9) Respiratory distress syndrome (RDS ) involves the formation of a hyaline membrane(hazy ground-glass appearance ches t radiograph) and a deficiency in surfactant.Measuring the lecithin-to-sphingomy elin ratio can help predict RDS. What is the major factor predisposing a newborn to RDS?a) Birth involving meconium b) Breech presentationc) Delayed cord clampi ngd) Pre-maturity (<34 wee s)e) Post-maturity (>45 wee s)10) What is the most co mmon cause of neonatal unconjugated hyperbilirubinemia?a) Post-birth UV-light ex posure b) Hemolytic disorder c) Bacterial sepsis Pediatrics 01May2009DO NOT DISTRIBUTE - 21 -d) Extrahepatic obstructione) Geneti c disorder (e.g. Rotor, Dubin-Johnson)11) Necrotizing enterocolitis (NEC) is see n in premature infants. Which of the followingis the most common cause of neonat al polycythemia?a) Birth involving meconium b) Breech presentationc) Delayed cor d clampingd) Pre-maturity (<34 wee s)e) Post-maturity (>45 wee s)12) A newborn h as a positive Coombs test, spherocytes, increased bilirubin, andincreased reticu

locytes. Which of the following is most li ely?a) Anemia of infancy b) Intravent ricular hemorrhagec) Immune hemolysisd) Hereditary spherocytosise) Glucose-6-pho sphate dehydrogenase deficiency13) A preterm infant is found to have bleeding of the germinal matrix, an area of immature vasculature that is the site of plurip otent cells that migrate to form neurons andglia. Which of the following is most li ely?a) Anemia of infancy b) Intraventricular hemorrhagec) Immune hemolysisd) Hereditary spherocytosise) Glucose-6-phosphate dehydrogenase deficiency14) What is the drug of choice for neonatal seizures?a) Phenobarbital b) Succinylcholin ec) Midazolamd) Fentanyle) Rohypnol15) All states require newborn screening for hypothyroidism. Which of the followingwould be suggestive of primary hypothyroid ism?a) Low T4 level and low TSH level b) Low T4 level and high TSH levelc) High T4 level and low TSH leveld) High T4 level and high TSH level16) Which of the fo llowing describes infants with Bec with-Wiedemann syndrome or islet cell adenoma s?a) Transient hypoglycemia b) Protracted hypoglycemiac) Transient hyperinsuline miad) Protracted hyperinsulinemia17) What is the most common type of tracheoesop hageal fistula (TEF)?a) Distal TEF with esophageal atresia b) H-type TEF without esophageal atresiac) Proximal TEF with esophageal atresia Pediatrics 01May2009DO NOT DISTRIBUTE - 22 -d) Proximal and distal TEF with esop hageal atresiae) Esophageal atresia without TEF (no air in GI tract)18) A double bubble sign on radiograph is indicative of:a) Distal TEF b) Proximal TEFc) Gast ric atresiad) Duodenal atresiae) Bile duct atresia19) What defect is seen here?a ) Gastroschisis b) Mec el diverticulumc) Hirschsprung diseased) Omphalocelee) Di aphragmatic hernia20) Failure of the tongue to descent by the 9th wee of gestat ion causes:a) Unilateral cleft lip b) Bilateral cleft lipc) Midline cleft palate d) Lateral cleft palate21) Which of the following are transient dar blue-blac pigmented macules over thelower bac and buttoc s that are sharply demar ed (do not fade into surrounding s in)?a) Child abuse bruises b) Erythema toxicum neona torumc) Seborrheic dermatitisd) Mongolian spots22) A newborn with IUGR is born w ith microcephaly and mental retardation. Physicalexam reveals mid-facial hypopla sia, micrognathia, and a flattened philtrum. Which of thefollowing did the mothe r li ely use during pregnancy?a) Cocaine b) Heroinec) Methamphetamined) Marijuan ae) Alcohol23) Infants of narcotic-abusing mothers should never be given naloxon e (Narcan) in thedelivery room as it can precipitate:a) Seizures b) Myocardial i nfarctionc) Stro ed) Respiratory distress syndromee) Ascites24) Multicystic idn ey is always unilateral and is the most common type of renaldysplasia. It consis ts of numerous non-communicating, fluid filled cysts, and a diagnosisis best con firmed by:a) MRI b) CT scanc) Abdominal x-ray Pediatrics 01May2009DO NOT DISTRIBUTE - 23 -d) Ultrasounde) Barium study25) What is the most common cause of hydronephrosis in childhood?a) Ureteropelvic juncti on obstruction b) Ureterovesical junction obstructionc) Autosomal dominate polyc ystic idney diseased) Autosomal recessive polycystic idney diseasee) Posterior urethral valves26) What is the treatment of choice for infants with recurrent U TIs caused byvesicoureteral reflex (VUR)?a) Nitrofurantoin b) TMP/SMXc) Amoxicil lind) Ciprofloxacine) Ceftriaxone27) What is the most common cause of end-stage renal disease in childhood?a) Ureteropelvic junction obstruction b) Ureterovesic al junction obstructionc) Autosomal dominate polycystic idney diseased) Autosom al recessive polycystic idney diseasee) Posterior urethral valves28) During an exam of an adolescent boy, the left side of the scrotal sac appears li e a bag o f worms. Palpation reveals tortuous veins. Which of the following is most li ely ?a) Varicocele b) Hydrocelec) Testicular torsiond) Hypospadiase) Cryptorchidism2 9) What is the most common pathogen seen in pediatric UTIs?a)S. aureus b)C. diff icilec) E. colid) H. influenzaee)S. saprophyticus30) A child is found to have ex treme proteinuria, hypoalbuminemia, hyperlipidemia, andedema. What is the most c ommon cause of primary nephrotic syndrome in the pediatric population?a) Minimal change disease b) Cystic renal dysplasiac) Membranous glomerulopathyd) Acute po st-streptococcal glomerulonephritise) Membranoproliferative glomerulonephritis31 ) A child presents with hematuria that is overt on microscopic examination andco ntains red cell casts. What is the most common glomerulonephritis seen in childh ood?a) Minimal change disease b) Cystic renal dysplasia Pediatrics 01May2009DO NOT DISTRIBUTE - 24 -c) Membranous glomerulopathyd) Acute

post-streptococcal glomerulonephritise) Membranoproliferative glomerulonephriti s32) A child presents with painless hematuria and sensorineural hearing loss. Da mage totype IV collagen is suspected (Alport syndrome). How is this disorder inh erited?a) Autosomal dominant b) Autosomal recessivec) X-lin edd) Y-lin ede) Mito chondrial33) Which of the following is the most common type of renal tubular aci dosis (RTA)seen in children and adults?a) Distal Type I b) Distal Type IVc) Prox imal Type II with Fanconi syndromed) Proximal Type II without Fanconi syndrome34 ) A patient is found to have a low serum pH and low serum bicarbonate. A wor up for hyperchloremic metabolic acidosis is begun. The patients urine anion gap (Na+ + K+ -Cl-) is found to be positive. The patient is hyper alemic and has a urine pH < 5.5. Whichof the following describes this patient?a) Normal b) Proximal RT A (type 2)c) Distal RTA (type 1)d) Distal RTA (type 4)e) None of the above35) Wh ich of the following would cause diabetes insipidus (DI)?a) Excess ADH release b ) Inability to release ADHc) Excess ACTH released) Inability to release ACTHe) H yperglycemia36) What is the most common form of acute renal failure (ARF) in the pediatric population?a) Pre-renal b) Intra-renal (intrinsic)c) Post-renal37) Wh at is the prophylaxis for preventing neural tube defects, which may be diagnosed by an increase in maternal alpha-fetoprotein?a) Niacin b) Diabetes controlc) El ectrolyte balanced) Sodium controle) Folic acid38) A newborn is found to have a tuft of hair on the lower bac . Testing shows anunderlying bony vertebral lesion without herniation of any spinal contents. Which of thefollowing is most li ely ? Pediatrics 01May2009DO NOT DISTRIBUTE - 25 -a) Myelomeningocele b) Meningocelec) Spina bifida occultad) Arnold-Chari malformation39) A child is found to have an Arnold-Chari malformation and begins developing signsof hydrocephalus. Which of the following is NOT a component of the Cushing triad?a) Upward gaze paralysis (setting sun sign) b) Bradycardiac) Hypertensiond) Cheyne-Sto es respiration40) Wh ich of the following is the term for bilateral lower extremity spasticity, which may be seen in cerebral palsy?a) Diplegia b) Hemiplegiac) Quadriplegiad) Seizur e41) A child is bought in for neurologic screening after the mother finds him bla n ingout for a few seconds. She says the boy will be tal ing, stop ta ing, then p ic bac up atthe same part of the sentence about 5 seconds later. EEG during on e of these episodesshows a characteristic 3-per-second spi e and wave pattern. W hich of the following ismost li ely?a) Generalized seizure b) Petit mal seizurec ) Infantile spasmd) Febrile seizure42) Reye syndrome is characterized by acute-o nset encephalopathy and degenerativeliver disease when what drug is given follow ing a viral illness (e.g. chic en pox)?a) Acetaminophen b) Aspirinc) Ibuprofend) Naproxene) Indomethacin43) A child presents with complaints of wea ness. Histor y reveals a viral illness about awee earlier. The wea ness has been progressive , ascending, and of acute onset. Testingreveals li ely autoimmune-mediated demye lination of peripheral nerves. Plasmapheresisis started to hasten resolution. Wh ich of the following is most li ely?a) Duchenne muscular dystrophy (DMD) b) Bec er muscular dystrophyc) Guillain-Barr syndromed) Myasthenia gravis (MG)e) Multipl e sclerosis (MS)44) Which of the following is an autoimmune disorder of the neur omuscular junctionwith autoantibodies binding to postsynaptic Ach receptors?a) D uchenne muscular dystrophy (DMD) b) Bec er muscular dystrophyc) Guillain-Barr syn drome Pediatrics 01May2009DO NOT DISTRIBUTE - 26 -d) Myasthenia gravis (MG)e) Multiple sclerosis (MS)45) A child presents with motor difficulties. Physical exam revea ls hypertrophy of thecalves. Watching the child stand up involves them pushing t heir trun up with their arms,then standing (Gower sign). An X-lin ed recessive disorder is suspected. Which of thefollowing is most li ely?a) Duchenne muscular dystrophy (DMD) b) Multiple sclerosis (MS)c) Guillain-Barr syndromed) Myasthenia gravis (MG)e) Spinal muscle atrophy (SMA)46) Which of the following primarily a ffects the anterior horn cell of the spine and hasmostly been eradicated due to a illed virus vaccination?a) Duchenne muscular dystrophy (DMD) b) SMA Type I (W erdnig-Hoffman disease)c) SMA Type IId) Poliomyelitise) Guillain-Barr syndrome47) Hydrocephalus is most associated with:a) Head size two standard deviations belo w mean (microcephaly) b) Head size one standard deviation below meanc) Abnormall y shaped head (craniosynostosis)d) Head size one standard deviation above meane)

Head size two standard deviations above mean (macrocephaly)48) Failure to thriv e (FTT) is defined as weight below what percentile?a) 10th b) 7thc) 5thd) 3rde) 1st49) Pediatric obesity is defined as weight above what percentile?a) 99th b) 9 7thc) 95thd) 93rde) 90th50) What is the most common cause of amblyopia in childr en?a) Retinal trauma b) Strabismusc) Retinoblastomad) Congenital cataracte) Reti nopathy of prematurity (ROP)51) What is the most common intraocular malignancy o f childhood, which presents withleu ocoria?a) Malignant melanoma b) Neuroblastom a Pediatrics 01May2009DO NOT DISTRIBUTE - 27 -c) Retinoblastomad) Metastatic adeno carcinomae) Ocular carcinoma52) What is the most common cause of leu ocoria in c hildren?a) Retinal trauma b) Strabismusc) Retinoblastomad) Congenital cataracte) Retinopathy of prematurity (ROP)53) Fluorescein drops with a blue-filter light would most li ely be used in which of thefollowing cases?a) Corneal abrasions b) Retinoblastomac) Congenital cataractsd) Retinopathy of prematuritye) Malignant melanoma54) Steroid drops should NOT be given in which of the following causes o f conjunctivitis?a) Neisseria gonorrhoeae b)Corynebacterium c)Streptococci d)St aphylococci e) HSV155) An infant presents with suspected developmental dysplasia of the hip (DHH). ABarlow maneuver is positive and an Ortolani maneuver confirm s the finding as the hip being abnormally positioned through most of the exam. W hich is most li ely?a) Subluxatable b) Dislocatablec) Dislocatedd) Fused56) Limp is the most common musculos eletal complaint in children. What is the mostcommo n cause of limp?a) Talipes equinovarus (clubfoot) b) Metatarsus adductusc) Genu valgumd) Genu varume) Trauma57) Legg-Calv-Perthes disease is defined as avascular necrosis of the:a) Navicular bone of the hand b) Lunate bone of the handc) Sacr oiliac jointd) Femoral heade) Talus bone of the an le58) Which of the following is NOT a contributing factor to slipped capital femorisepiphysis (SCFE)?a) Traum a Pediatrics 01May2009DO NOT DISTRIBUTE - 28 - b) Weightc) Male gender d) Puberty5 9) A young male child presents with painless limp and nee pain. Which of thefol lowing is most li ely?a) SCFE b) Duchenne muscular dystrophyc) Legg-Calv-Perthes diseased) DHHe) PCL tear 60) Which of the following is particularly common in to ddlers due to twisting forces onthe tibia during falling?a) Torus (buc le) fract ure b) Greenstic fracturec) Spiral fractured) Salter-Harris fracture type Ve) S alter-Harris fracture type IV61) What type of osteogenesis imperfecta (OI) is au tosomal dominant, has conductivehearing loss, blue sclerae, and presents with bo w legs or neonatal fractures?a) Type I b) Type IIc) Type IIId) Type IV62) A chil d is brought to the clinic by his mother after the childs father was swinging the child around the living room by his arms. The child holds his right arm close to the body,slightly flexing and pronating the hand. Motion at the elbow is limite d. Which of thefollowing is most li ely?a) Osteomyelitis b) Osteogenesis imperfe ctac) Lateral epicondylitisd) Medial epicondylitise) Radial head subluxation63) What is the most common pathogen seen in osteomyelitis in children?a)Shigella b )Salmonella c)S. aureus d)S. epidermidis e) N. gonorrhoeae 64) An adolescent pre sents with a painful nee, fever, and refusal to bear weight on theaffected limb . Aspiration reveals diplococci that grow on chocolate agar with carbondioxide, ruling out the most common cause. Which of the following is most li ely?a)Shigel la b)Salmonella c)S. aureus d)S. epidermidis e) N. gonorrhoeae Pediatrics 01May2009DO NOT DISTRIBUTE - 29 -65) What is the most common reason f or hospitalization in pediatric practice?a) Renal dysfunction b) Congenital hear t defectsc) Asthmad) Fever of un nown origin66) A child presents with acute resp iratory distress with dyspnea and wheezing. Examreveals subcostal retractions, t ripod positioning, tracheal tugging, and nasal flaring.Which of the following dr ugs would be most useful for this acute attac ?a) Beclomethasone (inhaled cortic osteroid) b) Flunisolide (inhaled corticosteroid)c) Epinephrine IVd) Albuterol ( beta2-agonist)e) Montelu ast (leu otriene receptor antagonist)67) A newborn pres ents with failure to thrive and pulmonary problems. An abdominalradiograph shows a mottle appearance, suggesting meconium ileus. Elevated levels of what ion wou ld be expected in this childs sweat?a) Potassium b) Sodiumc) Calciumd) Magnesiume ) Chloride68) A mother brings in her 2-month-old son to the pediatrician. The mo ther states thechild rolled off the sofa onto the carpeted floor. Testing shows

subdural hematomas.Physical exam reveals retinal hemorrhages. Which of the follo wing is most li ely?a) Caput succedaneum b) Cephalhematomac) Traumatic falld) Ch ild abusePediatrics #4 Extra: New Treatments For Multiple Sclerosis1) Which of t he following is NOT true of multiple sclerosis (MS)?a) Affects more than 30 per 100,000 in middle North America b) Has a higher prevalence in temperate zonesc) Most commonly affects women between ages 20 and 40d) In most cases, is caused by a viral infectione) Genetic factor may play a role in susceptibility2) Which of the following is characteristic of phase-2 MS, not phase-1?a) Hyperreflexic spa sticity b) Bladder and bowel affectedc) Hyperreflexia with good axial toned) Dim inution of deep tendon reflexese) Ataxia of gait with asymmetric paraparesis3) W hich of the following is characteristic of phase-2 MS, not phase-3?a) Bulbar dif ficulties with swallowing b) Gaze palsiesc) Frequency of urination and inability to completely void Pediatrics 01May2009DO NOT DISTRIBUTE - 30 -d) Bulbar difficulties with spea ing e) Limb and trun spasticity4) Which of the following during early-onset MS corr elates to a more severe course of disease?a) Length of initial event b) Loss of urinary controlc) Number of relapsesd) Intensity of initial evente) The appears to be no correlation between early-onset and disease course5) A young adult with MS has ocular pain, abnormal visual acuity and fields, reducedvision, relative afferent pupillary defect, and abnormal visual evo ed potentials (VEPs).Which of the following is most li ely?a) Uveitis b) Glaucomac) Retinoblastomad) Coats di seasee) Optic neuritis6) What is the central pathological CNS event suspected in the etiology of MS?a) Destruction of myelin b) Destruction of the putamenc) Des truction of the caudate nucleusd) Destruction of the substantia nigrae) Lysosoma l accumulation of lipids7) The T2-lesion volume (gadolinium-enhanced) on MRI at what locations may be anassociated predictor of depression, which occurs in 25-5 0% of patients with MS?a) Left anterior inferior prefrontal cortex and left ante rior temporal CSF b) Right anterior inferior prefrontal cortex and right anterio r temporal CSFc) Left anterior inferior prefrontal cortex and right anterior tem poral CSFd) Right anterior inferior prefrontal cortex and left anterior temporal CSF8) Which of the following tests is useful in evaluating demyelination in the posterior columns of the spinal cord?a) Visual evo ed response (VER) test b) Se nsory evo ed response (SER) testc) Rapid plasma reagin (RPR) testd) Erythrocyte sedimentation rate (ESR)e) Vitamin B12 levels9) The presence of gamma globulins with oligoclonal bands within the CSF indicates:a) The destruction of myelin b) The extent of future MS disease coursec) The synthesis of immunoglobulins in the CNSd) The destruction of autoantibodiese) An autoimmune reaction in the PNS10) What is the treatment for acute exacerbations of multiple sclerosis?a) Prednison e 500mg IV BID for 3-5 days b) Prednisone 100mg IV QID for 3-5 daysc) Prednisone 1000mg IM QD for 3-5 days Pediatrics 01May2009DO NOT DISTRIBUTE - 31 -d) Methylprednisolone 100mg IV BID f or 3-5 dayse) Methylprednisolone 1000mg IV QD for 3-5 days11) Which of the follo wing has been reported to increase in incidence in MS patientreceiving interfero n-beta?a) Headaches b) Influenza-li e symptomsc) Astheniad) Anemiae) Seizures12) Modafinil and amantadine are used in MS for patients complaining of:a) Spastici ty b) Depressionc) Insomniad) Fatiguee) Nausea13) 3,4-Diaminopyridine (3,4-DAP) may improve motor function and fatigue in some patients with MS. What is its mec hanism of action?a) Sodium channel bloc er b) Beta bloc er c) Potassium channel bloc er d) Calcium channel bloc er e) Alpha adrenergic agonist14) Baclofen and tizanidine are used in MS for patients complaining of:a) Spasticity b) Depressio nc) Insomniad) Fatiguee) Nausea15) Tolterodine may be used for hyperactive bladd er in MS patients. Which of thefollowing is a common side effect of this drug?a) Vomiting b) Dry mouthc) Bradycardiad) Conjunctival injectione) Profuse sweating 16) Which of the following is NOT part of the Barthof criteria in the diagnosis of MS,which requires 3 of 6 criteria to be met?a) Objective neurologic abnormali ties on clinical examination with involvementof white matter long tracts b) One area of CNS involvementc) Two or more episodes of progression over 6 monthsd) Ag e between 15 and 60 yearse) Exclusion of other diseases that may produce similar symptomsf) Presence of oligoclonal bands and/or myelin basic protein in CSF17) Which of the following disease-modifying drugs for MS has the highest cost and i

sdosed as 20mg SC once a day? Pediatrics 01May2009DO NOT DISTRIBUTE - 32 -a) Interferon beta-1a b) Interferon beta-1bc) Glatiramer d) Mitoxantronee) Natalizumab18) Which of the following has been shown to decrease the number and volume of gadolinium-enhancing lesions in MS patients, with a decline of about 44%?a) Botulinum-A toxin (Botox) b) Amanta dine (Symmetrel)c) Simvastatin (Zocor)d) Natalizumab (Tysabri)e) Tolterodine (De trol)19) Which of the following time periods shows an increase in MS relapses?a) Pre-pregnancy b) First trimester c) Third trimester d) First 3-months post preg nancye) 3 to 6-months post pregnancyPediatrics #5 Clinical: Textboo Questions1) A 2-year-old female child presents with VT, severe ventricular dysfunction,hypo tension, and metabolic acidosis. The patient is cardioverted into ventricular fi brillation, which degenerates into asystole. What is the most appropriate indica tion for using intravenous epinephrine in this patient?a) Ventricular ectopy b) Asystolec) Severe refractory metabolic acidosis and/or hyper alemiad) Bradycardi ae) Supraventricular tachycardia2) A 16-year-old female patient presents with sh ort stature and no secondary sexualcharacteristics. What diagnosis must be consi dered?a) Turner syndrome b) Isolated growth hormone deficiencyc) Cushing disease d) Familial short staturee) Addison disease3) Galactosemia, a disorder of carboh ydrate metabolism, is inherited in an autosomalrecessive fashion. What is the ri s of galactosemia in a child whose parents are bothcarriers for the disorder?a) 100% b) 75%c) 50%d) 25%e) 0%4) Which of the following statements is true regard ing children with sic le cell disease?a) Vaccinations are not required because t hey receive penicillin prophylaxis Pediatrics 01May2009DO NOT DISTRIBUTE - 33 - b) Gallstones typically develop bef ore the age of 3 yearsc) Episodes of dactylitis should be treated with antibioti csd) Hydroxyurea maintenance therapy decreases the number and severity of vaso-o cclusive crisese) Acute chest syndrome requires only supportive care5) A mother brings her 5-year-old son to your office in New Mexico for his regular healthmai ntenance visit. A quic review of the patient s chart reveals that he and his fa mily arestrict vegans. Their house is very small, so all the children spend a go od deal of timeoutside. The mother states that her son eats plenty of dar green vegetables and iron-fortified grains. She does not believe in providing supplem ental vitamins and minerals.This child is most at ris for nutritional deficienc y involving which of the following?a) Vitamin B12 b) Vitamin B6c) Niacind) Ribof lavine) Vitamin D6) A 6-year-old boy presents with a newly appreciated heart mur mur. He isasymptomatic, with normal growth and development and normal exercise t olerance. Onexamination S1 and S2 are normal; a II/VI low-frequency midsystolic murmur is heard atthe left lower sternal border. His pulses are normal. The most li ely diagnosis is:a) Bicuspid aortic valve b) Still s murmur c) Ventricular s eptal defectd) Atrial septal defecte) Coarctation of the aorta7) You are called to the delivery room for a routine birth. The infant cries when the cordis cut. You examine the child under the warmer and notice that when he stops crying, his chest heaves, and he turns blue. You are unable to pass the NG tube through the nose for suctioning. Which condition is most li ely causing this infant s respir atory distress?a) Choanal atresia or stenosis b) Vocal cord paralysisc) Subglott ic stenosisd) Recurrent laryngeal nerve damagee) Laryngeal web8) A 3-year-old gi rl is diagnosed with new-onset insulin-dependent diabetes mellitus.Which of the following laboratory findings is consistent with diabetic etoacidosis?a) Hypogl ycemia b) Hypercarbiac) Ketones in urined) Increased venous blood pHe) Decreased BUN9) During a male newborn examination, the testes are not palpable in the scr otal sacs.One testis is palpable high in the right inguinal canal and cannot be gently manipulatedinto the anatomically correct position. The left testis is not palpable but is discovered inthe abdomen after consultation with a pediatric ur ologist and an abdominal ultrasound. Incounseling the parents, which one of thes e statements regarding cryptorchidism is true? Pediatrics 01May2009DO NOT DISTRIBUTE - 34 -a) More than 99% of males have bilat eral descended testes at age 1 year b) Impaired sperm production is not a conce rn if neither testis descendsc) Malignant degeneration is not a ris factor for testes, which do not descend aslong as they are placed within the scrotal sac th rough surgery by 1 year of aged) This infant is no more li ely than his peers to

manifest an inguinal herniae) Microphallus is a common associated condition10) A 5-year-old boy presents with a waddling limp and has had a stiff right hip for thelast 2 months. He has minimal complaints of pain. The most li ely diagnosis is:a) Legg-Calve-Perthes disease b) Slipped capital femoral epiphysisc) Toddler s fractured) Septic arthritise) Juvenile idiopathic arthritis11) A 17-year-old y oung girl on oral contraceptive therapy for regulation of her menstrual periods presents with a 1-wee history of left leg pain and swelling. Evaluationwith a D oppler ultrasound reveals absence of flow in the left femoral and popliteal vein s.The clot extends proximally to the left external iliac vein. The most importan t potentialcomplication that one should be cautious about in this girl is:a) Ven ous insufficiency b) Limb overgrowthc) Pulmonary embolismd) Edemae) Gangrene12) A woman with a seizure disorder under medical management wants to conceive achil d. Her ris of having a child with a neural tube defect is greatest if her curre ntmedical regimen includes which of the following?a) Phenobarbital b) Phenytoinc ) Ethosuximided) Carbamazepinee) Primidone13) A 2-month-old infant presents to y our emergency department with a heart rate of 220 beats/minute, pulses, and adeq uate perfusion. After giving the infant oxygen, you noteabnormal P waves and a n arrow QRS (!0.08 sec) on the cardiac monitor. Which of thefollowing is the best course of action?a) Administer IV/IO epinephrine b) Administer IV adenosine by r apid bolusc) Administer IV calcium chlorided) Administer IV atropine by rapid bo luse) Administer IV sodium bicarbonate14) A 3-month-old infant presents with a h istory of abnormal movements that his parentsthin might be seizures. You observ e an episode of recurrent rhythmic flexor-extensor spasms that repeat about 30 t imes before subsiding. The EEG shows hypsarrhythmia, anda Wood lamp exam is posi tive for several flat, hypopigmented macules scattered over thes in surface. Thi s child s infantile spasms are most li ely a result of which of thefollowing und erlying disorders? Pediatrics 01May2009DO NOT DISTRIBUTE - 35 -a) Von Rec linghausen disease b) Tub erous sclerosisc) Von Hippel-Lindau diseased) Sturge-Weber diseasee) Bilateral a coustic neurofibromatosis15) A 21-month-old girl arrives at clinic in May with a vaccination record that indicatesthat she has received 3 DTaP doses, 3 Hib dose s, 3 IPV doses, 3 pneumococcal conjugatevaccine doses, 2 hepatitis A vaccine dos es, and 3 hepatitis B vaccine doses. Which of thefollowing should be administere d at this visit?a) DTaP, Hib, IPV, varicella b) DTaP, Hib, pneumococcal conjugat e vaccine, MMR, and varicellac) DTaP, hepatitis A, IPV, pneumococcal conjugate v accined) DTaP, hepatitis B, MMR, and varicellae) DTaP, hepatitis A, IPV, MMR, an d varicella16) The mother of a 30-month-old boy is concerned that the child s sp eech is garbled.The child uses ma-ma and da-da appropriately. He uses about 30 other w ords, butmost of them are mispronounced (for instance, boo instead of blue). The boy s aunt,uncle, and cousins came to visit for a wee end and were unable to unders tand more thanhalf of what he said. Examination of the ears reveals normal canal s with translucent,mobile tympanic membranes, and visible landmar s. Which of th e following evaluationsfor speech delay should be performed first?a) Receptive l anguage testing b) Phonetic testingc) Dysfluency evaluationd) Tympanogram testin ge) Audiologic (hearing) assessment17) A 13-year-old girl presents with recurren t abdominal pain over the last 3 months. Shehas missed a total of 8 days of scho ol. There is no associated fever, weight loss,gastrointestinal bleeding, and the pain does not occur in relation to meals or awa en her from sleep. There is dif fuse abdominal tenderness but no other abnormal findings onexamination. Which ap proach is li ely to help in the diagnosis and management of her condition?a) Abd ominal CT scan with contrast b) Upper and lower endoscopy and biopsiesc) Explain ing the li ely etiology of her symptoms using a biopsychosocial modeland symptom atic therapyd) A diet history and a diet elimination triale) Referral to a psych iatrist18) A newborn male child has a flat facial profile, upslanted palpebral f issures, epicanthalfolds, a small mouth with a protruding tongue, small genitali a, and simian creases on hishands. What of the following chromosomal disorders i s most li ely in this child?a) Trisomy 21 b) Trisomy 18c) Trisomy 13d) Klinefelt er syndromee) Turner syndrome Pediatrics 01May2009DO NOT DISTRIBUTE - 36 -19) At a 2-year well-child visit, yo u collect information that your patient lives in a veryold rental home with peel

ing paint. Both the capillary (screening) and venous blood leadmeasurements are 50"g/dL. The patient has a history of constipation but is otherwiseasymptomatic. Which of the following courses of action is most appropriate?a) Initiate chelat ion therapy in a lead-free environment within 48 hours b) Redraw the blood lead level in 1 wee and test all siblings; treat if # 50"g/dLc) Optimize calcium and iron inta e and repeat the blood lead level in 1 month;treat if # 50"g/dLd) Refer the family to a lead-removal company; repeat the blood lead leve l 1month after decontamination of the home, and treat if # 50"g/dLe) Refer the case to child protective services for parental neglect20) A young couple is in your office for their prenatal visit, and you are discussing infantfeeding. The father states that he prefers that the mother breastfeed the baby. The mother is hesitant to commit to breastfeeding because she plans on ret urning to full-timeemployment 6 wee s after the child is born. Neither her mothe r nor her sisters chose to breastfeed. She is concerned that human breast mil m ay not provide all the nutrients thatthe child needs, and she believes formula i s a more complete nutritional source for infants. She is willing to consider exc lusive breastfeeding based on the AmericanAcademy of Pediatrics recommendation. If her baby is exclusively breastfed, whenshould the child begin receiving oral vitamin D supplementation?a) Never b) Within the first month of lifec) Age 2 mo nthsd) Age 4 monthse) Age 6 months21) A 12-year-old female patient presents with fever, night sweats, weight loss, fatigue,anorexia, and painless, rubbery, cerv ical lymphadenopathy. What is the most common presentation of Hodg in disease?a) Fever, night sweats, and/or weight loss of >10% in the preceding 6 months b) Me diastinal lymphadenopathyc) Painless, rubbery, cervical lymphadenopathyd) Prurit use) Extreme fatigue and anorexia22) Which of the following medication groupings is most appropriate for a patient 12years old with persistent asthma who has fa iled to achieve well-controlled asthma whilereceiving step 2 treatment?a) None b ) A daily low-dose inhaled corticosteroidc) A daily medium-dose inhaled corticos teroid and a long-acting inhaled$-2-agonistd) A daily low-dose inhaled corticost eroid and a long-acting$-agoniste) A daily medium-dose inhaled corticosteroid an d nedocromil23) Crops of papular, vesicular, pustular lesions starting on the tr un and spreading to theextremities, in addition to small, irregular red spots w ith central gray or bluish-whitespec s that appear on the buccal mucosa, is the classic description of which of thefollowing infections? Pediatrics 01May2009DO NOT DISTRIBUTE - 37 -a) Measles b) Erythema infectiosum ( fifth disease)c) Roseola infantumd) Zoster (shingles)e) Rubellaf) Hand-foot-mout h diseaseg) Chic enpox24) A 20-month-old boy who was treated with high-dose amox icillin (90mg/ g per day)for acute otitis media 3 wee s ago now presents with ac ute-onset ear pain, a bulging,erythematous right tympanic membrane, and decrease d mobility on pneumatic otoscopyexamination. Which of the following is the most appropriate antibiotic for this child?a) Azithromycin b) Amoxicillin-clavulanate c) Erythromycind) Trimethoprim-sulfamethoxazolee) Dicloxacillin25) Which of the following is considered a ris factor for neonatal respiratory distresssyndrome? a) Neonatal sepsis b) Poorly controlled maternal diabetesc) Maternal preeclampsi ad) Neural tube defectse) Trisomy 2126) A mildly febrile 6-year-old patient pres ents to your office with dysuria and urinaryfrequency and urgency. She has a his tory of one prior UTI about 8 months ago. Youobtain a dipstic urinalysis and se nd a urine culture. The dipstic is positive for nitritesand leu ocyte esterase. Which of the following is the most appropriate course of action atthis time?a) Await culture results and tailor therapy based on bacterial sensitivities b) Beg in empiric amoxicillinc) Begin empiric amoxicillin and schedule the child for a renal ultrasound withinthe next 6 wee sd) Begin empiric amoxicillin and schedule the child for a renal ultrasound andvoiding cystourethrogram within the next 6 wee se) Admit the child to the hospital for IV ampicillin and gentamycin and sch edule aDMSA scan27) A 3-month-old infant presents with cyanosis and an echocardi ogram reveals that thechild has tetralogy of Fallot. What four associated lesion

s describe tetralogy of Fallot?a) Ventricular septal defect, over-riding aorta, pulmonary stenosis, rightventricular hypertrophy b) Ventricular septal defect, a trial septal defect, pulmonary stenosis, rightventricular hypertrophyc) Ventricu lar septal defect, atrial septal defect, aortic stenosis, right ventricular hype rtrophyd) Ventricular septal defect, coarctation of the aorta, aortic stenosis, rightventricular hypertrophy Pediatrics 01May2009DO NOT DISTRIBUTE - 38 -e) Ventricular septal defect, mitral valve prolapse, pulmonary stenosis, leftventricular hypertrophy28) A 3-year-old boy with a nown diagnosis of factor XI deficiency presents to theemergency dep artment with uncontrolled bleeding from a lip laceration following a fall.The mo st appropriate product that can be used for factor replacement in this child pri or tosuturing is:a) Cryoprecipitate b) Granulocyte infusionsc) Fresh frozen plas ma (FFP)d) Platelet transfusione) DDAVP29) At the health maintenance visit for a 12-year-old male, you note that he has enteredhis pubertal height growth spurt. The patient s mother as s about what changes her sonshould be expecting in his body over the next several years. As part of your review, youmention that the mo st typical sequence of pubertal events in males is which of thefollowing?a) Pea height velocity, pubarche, penile enlargement, testicular enlargement b) Pea h eight velocity, testicular enlargement, penile enlargement, pubarchec) Testicula r enlargement, pubarche, penile enlargement, pea height velocityd) Testicular e nlargement, pea height velocity, penile enlargement, pubarchee) Pubarche, testi cular enlargement, pea height velocity, penile enlargement30) A 4-year-old chil d with nown asthma presents to the emergency department with achief complaint o f wheezing for the past 8 hours. On examination he is alert andcooperative, mild ly tachypneic, has diffuse loud expiratory wheeze, and has a pulseoximetry readi ng of 89% while breathing room air. He has already ta en 3 albuterolaerosols at home in the past hour. He is unchanged after receiving another albuterolinhalati on treatment in the emergency department. Appropriate next management wouldinclu de:a) Supplemental oxygen b) Albuterol inhalationc) Ipratropium bromide inhalati ond) Oral corticosteroidse) All of the above31) A previously healthy 3-year-old boy presents with a history of fever and diarrhea for the past 2 days. The fever has not responded to ibuprofen, and his urine output hasdecreased today. On exa mination, he is alert, has a temperature of 101F, heart rate of 115 beats per min ute, blood pressure of 105/60 mm Hg, and mild diffuse abdominaltenderness. The s erum electrolytes are normal, but his BUN is 60 mg/dL and his serumcreatinine is 1.8 mg/dL. The complete blood count is normal. Urinalysis shows 1+ protein, sma ll blood, and occasional hyaline casts. The idney ultrasound is normal.Which of the following statements regarding his acute renal failure is most accurate?a) It is due to hemolytic-uremic syndrome b) It is due to pyelonephritisc) It is du e to interstitial nephritisd) It is due to the use of ibuprofen in a dehydrated statee) It is due to urinary tract obstruction Pediatrics 01May2009DO NOT DISTRIBUTE - 39 -32) A 14-year-old girl presents with several wee s of profound fatigue, intermittent low-grade fevers, a facial rash , and joint pain. The rash recently worsened mar edly after sunexposure. On phys ical examination, she has a malar rash extending over the bridge of thenose, but sparing the nasolabial folds, painless oral ulcers, and painful limitation of m ovement in her wrists and finger joints. On laboratory testing, her WBC is 3,500 /mm3,Hgb 9.5 g/dL, platelet count 120,000/mm3. A urinalysis shows 15 to 19 RBC/h pf and anelevated protein of 100 mg/dL. Which of the following tests will most l i ely be positive?a) Antinuclear (ANA) antibody b) Rheumatoid factor (RF)c) Anti -double-stranded DNA (dsDNA) antibodyd) Anti-Smith (Sm) antibodye) Anti-Ro (SS-A ) antibody33) A 3-month-old female infant presents to your emergency department unresponsiveand with fever, tachypnea, bradycardia, and hypotension. What order should you followin your initial assessment?a) Airway, breathing, circulation, d isability, exposure b) Breathing, airway, circulation, disability, exposurec) Ci rculation, airway, breathing, exposure, disabilityd) Exposure, breathing, airway , circulation, disabilitye) Exposure, airway, breathing, circulation, disability 34) A 4-year-old male child presents with abrupt-onset petechiae and ecchymoses. Other than the s in findings, the child appears well and is hemodynamically sta ble. Nosplenomegaly is noted. A complete blood count reveals a normal white bloo

d cell count,a normal hematocrit, and a platelet count of 12,000 per mm3. Large platelets are seen onthe peripheral smear. No premature white cell forms are see n on peripheral smear. The parent reports that the child had a viral illness 2 w ee s before presentation. Which of thefollowing is the most li ely diagnosis?a) Isoimmune thrombocytopenia b) Leu emiac) Sepsisd) Immune thrombocytopenic purpur ae) Hypersplenism35) A child presents with a reduced number of CD3+ T cells, an increased number of Blymphocytes that are mildly abnormal in function, has a con otruncal heart lesion,hypoplastic thymus, and hypocalcemia. Which of the followi ng chromosomal disorders ismost li ely in this child?a) Zellweger syndrome b) Mi crodeletion of 22q11.2c) Trisomy 13d) Gaucher diseasee) Wilson disease36) A 4-mo nth-old former 30-wee premature infant is seen in late October for well-childca re. His mother is concerned about the transfusions that the infant required duri ng her course in the neonatal intensive care unit and wishes to restrict her exp osure to blood products. Referral for administration of which of the following w ould be mostappropriate to limit her ris of severe bronchiolitis? Pediatrics 01May2009DO NOT DISTRIBUTE - 40 -a) Ribavirin b) Nasal influenza vacc inec) Injected influenza vaccined) IV RespiGame) IM palivizumab37) The mother of a 2-month-old infant brings her daughter to your office during thesummer for he r regular health maintenance visit. The child is cared for by her maternalgrandm other 3 days a wee while the mother is at wor . The infant is exclusively fed a cow mil -based commercial formula when she is with the mother; the grandmother believes that the child should also receive juice diluted with water due to the warmweather. Which of the following represents the most appropriate dietary coun selingregarding this infant s diet?a) Formula-fed infants at this age require fr ee water supplementation during warmmonths to maintain optimal hydration b) Form ula-fed infants at this age require glucose supplementation during thewarm month s to maintain optimal caloric inta ec) Formula-fed infants do not require any ad ditional vitamin, mineral, caloric, or fluid supplement beyond their formula for the first 6 months of lifed) Dilution of this infant s formula with water or ju ice on the days that she is withthe maternal grandmother is unnecessary but harm lesse) This infant should be switched to a soy protein-based formula38) No red r eflex is seen on fundoscopic examination of a newborn. Which of thefollowing is the most li ely diagnosis?a) Retinoblastoma b) Leu ocoriac) Congenital cataractd ) Congenital glaucomae) Toxocariasis39) A 5-year-old boy is brought to your offi ce complaining of progressive fatigue,wea ness, and nausea over the past few mon ths. He was a model student, but he is nowhaving trouble in school and displayin g frequent outbursts, the last of which resulted inhis being sent home for hitti ng another child. Initial lab results show mild hypoglycemia,hyponatremia, and h yper alemia. The child is diagnosed with adrenal insufficiency andtreated approp riately; however, his behavior continues to worsen, and he begins to havedifficu lty wal ing and spea ing. Which of the following is the most li ely etiology of his behavior problems?a) Tay-Sachs disease b) Gaucher diseasec) Niemann-Pic dis eased) Adrenoleu odystrophye) Rett syndrome40) An 8-year-old girl thought to hav e attention-deficit disorder (inattentive-type)undergoes EEG testing and is foun d to have a 3-Hz spi e-and-wave pattern. Results of the EEG, coupled with videot aping of episodes of the patient s inattention, lead to adiagnosis of childhood ab sence epilepsy. Which of the following is most appropriate for initial treatment of the child s disorder? Pediatrics 01May2009DO NOT DISTRIBUTE - 41 -a) Methylphenidate b) Carbamazepinec ) ACTHd) Ethosuximidee) Phenobarbital41) A child presents with lymphedema of the hands and feet, a shield-shaped chest,widely spaced hypoplastic nipples, short stature, and multiple pigmented nevi. Inaddition, she had a coarctation of the a orta that was repaired and has renal disease. Her parents continue to be worrie d that there is something in addition to her heart conditionthat is causing fail ure to thrive. Which of the following chromosomal disorders is mostli ely in thi s child?a) Trisomy 21 b) Trisomy 18c) Trisomy 13d) Klinefelter syndromee) Turner syndrome42) A 14-year-old patient familiar to the emergency room staff due to m ultiple visits inthe last 3 months is brought in by her mother for ingestion of an un nown number of acetaminophen tablets. The mother states that she eeps all the medicines in the houseloc ed up because this is just the sort of thing my da

ughter would do to me. She sawthe girl stuffing something into her bedside drawer while she was passing the girl s roomand discovered a bottle mar ed acetaminophe n, 250 tablets. Only 4 tablets remained inthe bottle. The mother did not believe that the daughter too the tablets until she beganvomiting about an hour later. The girl refuses to spea in her mother s presence buteventually admits that she too many tablets about 4:00 p.m. (3 hours ago). Which of the following is recomm ended as an antidote for this patient s ingestion?a) Atropine sulfate b) Hemodia lysisc) Whole bowel irrigationd) Oral N-acetylcysteinee) Activated charcoal43) W hich of the following is consistent with abuse rather than accidental injury?a) A 30-month-old child with a buc et handle fracture b) A 12-month-old infant with a rib fracturec) A 6-month-old infant with retinal hemorrhages in the absence o f signs of external head traumad) Abdominal bruises in a 9-month-old infante) Al l of the above44) A 2-year-old presents with painless rectal bleeding. The hemog lobin is 9 g/dL.Capillary refill remains normal. The best next step to positivel y identify the cause of bleeding is:a) Colonoscopy b) Transfusion with pac ed r ed blood cellsc) Mec el diverticulum scand) Gastric lavagee) Stool culture Pediatrics 01May2009DO NOT DISTRIBUTE - 42 -45) A 3-wee -old male infant present s to the emergency department with 24-hour historyof vomiting and poor feeding. He is found to be hypotensive and hypoglycemic. Hisserum electrolyte values are as follows: Na 121mmol/L, K 6.9mmol/L, CO2 20mmol/L,chloride 105mmol/L, BUN 17 m g/dL, creatinine 0.7mg/dL, and glucose 36mg/dL. Hereceives 20mL/ g NS fluid bolu s and 2mL/ g dextrose 25. What other life-savingintervention should this infant receive?a) IV azithromycin b) IV bicarbonatec) IV hydrocortisoned) IV albumine) IV calcium46) A 7-year-old girl presents with a 3-wee history of dozens of asym ptomatic red, scaly5 to 10 mm plaques appearing on the trun . When the scales ar e pulled off, they bleed.Her nails are pitted. The most appropriate laboratory t est is:a) A bacterial culture of the red plaques b) A fungal culture of the red plaquesc) A throat cultured) A Tzanc smear e) A complete blood count47) An 8-ye ar-old patient of yours with attention-deficit/hyperactivity disorder isexperien cing unacceptable adverse effects due to his stimulant medication. You have pres cribed immediate- and extended-release preparations of two separate agents in th e past. You believe that the patient may benefit from switching to a non-stimula ntmedication. Which of the following medications approved for the treatment of a ttention-deficit/hyperactivity disorder is classified as a non-stimulant?a) Oral atomoxetine b) Oral lisdexamfetaminec) Oral methylphenidated) Oral dextroamphet aminee) Oral mixed amphetamine salts48) A 9-month-old girl presents with a 3-day history of fever to 103F (39.4C). Thismorning, the girl developed a rash. On phys ical examination, the girl is afebrile and hasan erythematous, maculopapular ras h over her trun , arms, and legs. Which of thefollowing is the most li ely cause of this patient s illness?a) Human parvovirus B19 b) Measlesc) Human herpesviru s 6d) Chic enpoxe) Group A beta-hemolytic streptococci49) A 2-year-old child is brought to the emergency department following a brief (<2minutes) generalized se izure. Initial vitals include a temperature of 102.9F. Followingthe history, phys ical examination, and laboratory studies, you determine that the patienthas had a febrile seizure. The parents are appropriately concerned and have a number of questions. You would be correct in telling them which of the following? Pediatrics 01May2009DO NOT DISTRIBUTE - 43 -a) Children who experience a single febrile seizure have no greater ris of subsequently developing epilepsy than ch ildren who have not experienced afebrile seizure b) The morbidity and mortality associated with febrile seizures is extremely highc) At least half of patients w ho experience an initial febrile seizure willexperience seizures with subsequent episodes of fever d) Patients who have experienced a single febrile seizure sho uld be placed on preventative anticonvulsant medicatione) Febrile seizures are u sually associated with intracranial infections50) A 3-year-old boy presents with an elbow hemarthrosis after falling on his elbow.There is no history of spontan eous bleeding. There is no history of epistaxis, gingival bleeding, or cutaneous bruising. The child s maternal grandfather had frequentspontaneous bleeding and hemarthroses after trauma on multiple occasions. Laboratoryresults revealed a p rolonged PTT, normal PT, and a platelet count of 150,000 per mm3.The factor VIII coagulant activity (VIII:c) is low and the factor IX level is normal. Whatis th

e most li ely diagnosis?a) Idiopathic thrombocytopenic purpura b) Von Willebrand diseasec) Vitamin K deficiencyd) Hemophilia Ae) Liver disease51) A 10-wee -old boy is brought to the emergency department by his mother with ahistory of failur e to thrive and poor feeding. He occasionally vomits small amounts of formula. H is birth weight, length, and head circumference were at the 50th percentile;howe ver, his weight has dropped to the 10th percentile and his length to between 25t hand 40th percentiles. His vital signs are normal, and the physical exam is othe rwiseunrevealing. Venous blood gas and electrolyte study results include: pH 7.3 2, sodium 134mEq/L, potassium 4.5 mEq/L, chloride 106 mEq/L, and bicarbonate 10 mEq/L. Which of the following diagnoses is the most li ely?a) Inborn error of me tabolism b) Renal tubular acidosisc) Pyloric stenosisd) Chronic diarrheae) Cysti c fibrosis52) A 2-year-old girl presents with a swollen left nee, limping, and morning stiffness inthe left nee of 3 months duration. On physical examination , there is a left nee jointeffusion, synovial thic ening, and limitation of mov ement. In addition, the left leg islonger than the right and there is atrophy of the quadriceps. The remainder of the reviewof systems and physical examinations is normal. On laboratory testing, a complete bloodcount is normal. An antinucle ar (ANA) antibody test is positive at a titer of 1:320. Thischild is at most ris for which of the following sequelae/complications?a) Glomerulonephritis b) Hem olytic anemiac) Chronic, non-granulomatous anterior uveitis (iridocyclitis)d) Ac ute anterior uveitis (iridocyclitis)e) Rheumatic heart disease Pediatrics 01May2009DO NOT DISTRIBUTE - 44 -53) A 9-year-old boy diagnosed with pneumonia 2 days ago presents to the emergencydepartment via ambulance in respir atory distress. His past medical history isnoncontributory, and he is at low ris for contracting tuberculosis. He is hypoxic andrequires oxygen. A STAT portabl e chest radiograph reveals a large right-sided pleuraleffusion, which shifts in the decubitus position. Fluid is obtained via thoracentesis for Gram stain and c ulture. Which of the following is the most li ely pathogen responsiblefor this b oy s pneumonia?a) Staphylococcus aureus b) Nontypeable Haemophilus influenzaec) Chlamydophila pneumoniaed) Klebsiella pneumoniaee) Mycoplasma pneumoniae54) Duri ng a routine annual physical examination, a 9-year-old previously healthy girlha s a blood pressure of 140/75 mm Hg in all four extremities. The physical examina tionis otherwise completely normal, except for obesity. The family history is po sitive for hypertension in the father and paternal uncle. The blood pressure rem ains in the 140/70mm Hg range on two repeat examinations performed 1 wee apart, using a cuff that isappropriate for her obesity. The urinalysis, serum electrol ytes, and serum creatininelevels are normal. Which of the following is the most appropriate next step in themanagement of this patient?a) Reassure the patient t hat her blood pressure is normal for her size b) Advise observation, with repeat blood pressure chec s every monthc) Advise an immediate evaluation by a nephrol ogist and cardiologistd) Advise a regimen of weight reduction and regular exerci see) Advice a regimen of diuretic therapy55) A parent brings her 12-wee -old chi ld to your office because he has a scaly facialrash. The boy was exclusively bre astfed for 8 wee s but was switched to commercial cowmil -based formula about a month ago when his mother went bac to wor . She has been putting lotion on the rash, but it has not helped. The child s birth weight was at the 50th percentile but has now dropped toward the 25th percentile line. The physical examinationre veals an eczematous rash over both chee s. The stool is guaiac-positive but not grossly bloody. Based on the history and physical examination, you suspect that the patient may be allergic to cow mil protein. Which of the following is the b est next step in themanagement of this patient?a) Recommend that the mother see her obstetrician about medication to help her begin lactating again b) Switch t he patient from cow mil -based formula to whole cow s mil c) Switch the patient from cow mil -based formula to soy formulad) Switch the patient from cow mil -b ased formula to a protein hydrolysateformulae) Begin parenteral alimentation to permit total bowel rest56) A 16-year-old male is brought to your office by his m other, who insists that you perform a urine drug screen on her son. You begin by interviewing the mother and theyoung man together, but explain to the parent th at you will also be conducting part of theinterview and the physical examination without her present in the room. She states thatshe will only agree to let you

spea with him alone if you agree to discuss with her any Pediatrics 01May2009DO NOT DISTRIBUTE - 45 -high-ris behaviors that he admits t o engaging in. Concerning patient confidentiality inregard to adolescents, you a re required by law to inform the parent of this minor of whichof the following?a ) Use of marijuana b) Suicidal ideationc) Petty theftd) Consensual sexual relati ons with another minor of the opposite gender e) Consensual sexual relations wit h another minor of the same gender 57) You see a 4-year-old child for declining school performance and behavior problems.His mother notes that he is a poor slee per. He snores loudly and often gasps in his sleep.Sometimes she sleeps with him because she is afraid he will stop breathing. You note aslight fall off the gro wth curve and very large tonsils. A nec film demonstrates largeadenoids as well . The child s insurance company will not pay to have the tonsils andadenoids rem oved unless you can prove they are causing him significant health problems.Which test is the most li ely to give you that information?a) Bronchoscopy b) Overnig ht pulse oximetry monitoringc) Polysomnographyd) Fluoroscopye) Overnight EEG mon itoring58) An infant who was discharged from the hospital on day 2 of life prese nts to your office 3 days later for follow-up. The mother did not receive prenat al care. You notice bilateral purulent discharge from the eyes. There is mar ed eyelid edema andconjunctival swelling (chemosis). What is the most li ely pathol ogic agent?a) Chlamydia trachomatis b) Neisseria gonorrhoeaec) Group B Streptoco ccusd) Toxoplasma gondiie) Treponema pallidum59) An unresponsive adolescent pati ent is brought to the emergency department withsuspected ingestion of an un nown substance. EMS received a call from the hotel roomwhere the youth was found, bu t no one else was there when they arrived. The patient ison 100% inspired oxygen and has required several bouts of positive pressure ventilationin the ambulance . On exam, the patient has a heart rate of 55, blood pressure 85/50, pinpoint pu pils, and trac lines on his left arm. Along with ongoing cardiovascular andresp iratory support, which of the following should be administered to this patient?a ) Pralidoxime chloride b) Physostigminec) Naloxoned) Atropine sulfatee) Deferoxa mine60) A 15-month-old boy is brought to the emergency department with a fever a nddifficulty breathing. Right-sided wheezing is noted on the physical examinatio n. The patient does not improve with aerosolized nebulizer treatment. An inspira tory chestradiograph is normal; however, the expiratory film demonstrates rightsided Pediatrics 01May2009DO NOT DISTRIBUTE - 46 -hyperinflation, with mediastinal shi ft to the left. This patient s respiratory symptoms aremost li ely due to which of the following?a) Pneumonia b) Foreign body aspirationc) Pneumothoraxd) Empyem ae) Viral upper respiratory infection61) You are seeing an 18-month-old boy who is new to your practice. His father isconcerned about his child s development in relation to his two older brothers. The boyavoids eye contact and does not resp ond to efforts to engage him in reciprocal play suchas pee -a-boo and patty ca e games. He does not generate spontaneous language but canrepeat certain words if spo en to him over and over. He spends a lot of time by himself roc ing bac an d forth and becomes very agitated if this activity is interrupted. Which of the following conditions is most consistent with this child s reported behaviors?a) Down syndrome b) Hearing impairmentc) Autismd) Attention-deficit/hyperactivity d isorder e) Asperger syndrome62) An 8-year-old boy is referred to the emergency d epartment by his pediatrician for achief complaint of wea ness. The wea ness has been slowly progressive over the lastseveral wee s. A review of symptoms reveal s a history of constipation, polyuria, and polydipsia. The child is on no medica tions, and past medical history is noncontributory.In the primary physician s of fice, the patient had a serum potassium measurement of 2.8mEq/L. A blood pressur e measurement in the emergency department is normal for age,height, and gender. Urine electrolyte studies reveal an elevated urine potassium value.Which of the following conditions is the most li ely cause of this patient s hypo alemia?a) E xcessive sweating b) Renal tubular acidosisc) Anorexia nervosad) Cushing syndrom ee) Renovascular disease63) A 2-wee -old female infant presents with generalized hypotonia, duodenal atresiaand hypothyroidism. What other structural defect is she most li ely to have?a) Malrotation b) Endocardial cushion defectsc) Cleft pa lated) Renal diseasee) Sensorineural hearing loss64) Which of the following cond

itions are often associated with polyhydramnios?a) Duodenal atresia b) Tracheoes ophageal fistulac) Congenital hydrocephalus with myelomeningoceled) Renal agenes ise) A, B, and C Pediatrics 01May2009DO NOT DISTRIBUTE - 47 -65) A 3-year-old boy presents with v iolent episodes of intermittent colic y pain, emesis,and blood per rectum. A tub ular mass is palpated in the right lower quadrant. Theabdominal radiograph revea ls a dearth of air in the right lower quadrant and air-fluidlevels consistent wi th ileus. Which of the following procedures will best assist indiagnosis and tre atment?a) Esophagogastroduodenoscopy b) Rectal biopsyc) Air contrast or double c ontrast enemad) Stool culturee) Colonoscopy66) An 18-month-old female child pres ents with blood-strea ed stool. The stool isgrossly positive on occult blood tes ting. Which of the following diagnoses is most li ely?a) Anal fissure b) Peptic ulcer diseasec) Mallory-Weiss tear d) Inflammatory bowel diseasee) Necrotizing e nterocolitis67) A 4-year-old boy was seen by his pediatrician for fever and abdo minal pain. The pain began after a sledding accident the day before his visit in which he fell on his right side.His mother noticed that his abdomen appeared di stended today, particularly on the rightside. In the pediatrician s office, he i s noted to be hypertensive and has gross hematuria.What is the most li ely diagn osis?a) Pyelonephritis b) Liver contusionc) Renal contusiond) Wilms tumor e) Neu roblastoma68) You are called to evaluate a newborn with an apparent foot deformi ty. On closeexamination, you note adduction of the forefoot, inversion of the fo ot, and plantar flexionat the an le that is relatively fixed. Which of the follo wing is true of this patient scondition?a) This clinical picture is most consist ent with metatarsus adductus b) This deformity will respond to stretching exerci sesc) This deformity will correct spontaneously when the child is able to bear w eightd) This deformity will require surgical repair e) This deformity may be ass ociated with other congenital malformations69) A 12-year-old boy with Crohn dise ase for 2 years is seen with an acute exacerbation.He is complaining of abdomina l pain and diarrhea and has right lower quadrant fullness.The most effective app roach in this acute setting is which of the following?a) Perform a colonoscopy f or cancer surveillance b) Obtain a stool culture and to exclude acute infectious colitis and imagingstudies to evaluate for abscess or fistulac) Initiate therap y with mercaptopurine or azathioprined) Perform a capsule endoscopye) Start biol ogic therapy with anti-TNF alpha antibody Pediatrics 01May2009DO NOT DISTRIBUTE - 48 -70) A 3-year-old girl periodically e xperiences swelling around her lips and brea s out inhives when she eats the sna c s provided at daycare. Which of the following is the mostappropriate for deter mining whether the child s symptoms are due to food allergies?a) S in pric test ing to foods b) Food-specific IgE levelsc) S in pric testing to foods followed by double-blind placebo-controlled foodchallengesd) Open-label food challengese) Endoscopy71) An 11-year-old girl is referred to your office following an abnorm al screen for scoliosis. You diagnose idiopathic scoliosis on exam using Adam s forward bending test.Subsequent radiographs reveal a lateral curvature of 35-deg rees. The patient is pre-menarchal. You refer the patient to an orthopedic surge on and counsel the parent that thespecialist will probably recommend:a) External bracing b) Follow-up radiographs every 6 monthsc) Stretching exercisesd) Surgic al fixatione) No intervention72) A child in the emergency department has point t enderness over the proximal tibia andan appropriate history of trauma. The radio graph shows a fracture through the growth plate that extends into the epiphysis and joint space. This type of fracture would becharacterized as:a) Salter-Harris Type I b) Salter-Harris Type IIc) Salter-Harris Type IIId) Salter-Harris Type I Ve) Salter-Harris Type V73) A 4-year-old Caucasian boy presents for evaluation o f persistent jaundice. The familyreports that the boy had neonatal jaundice on t he first day of life, and was treated with phototherapy. He has always had mild icterus, but has had increased icterus at times,especially following other mild illnesses, such as ear infections and colds. There is afamily history of his fat her and paternal grandmother having undergone splenectomy. Onexamination, the bo y has mild scleral icterus, and his spleen is palpable about 3 cm belowthe left costal margin. The laboratory evaluation reveals a total bilirubin of 1.9 mg/dL( unconjugated fraction is 1.5 mg/dL), normal liver transaminases, hemoglobin of 1

1.2gm/dL, a normal MCV, and an elevated reticulocyte count of 8%. An osmotic fra gilitytest is performed and demonstrates positive results. What is the most li e ly diagnosis?a) Iron-deficiency anemia b) Hereditary spherocytosisc) Acute blood lossd) Acute leu emiae) Sic le cell disease74) An adolescent comes to you with a chief complaint of painless vaginal discharge.You note projection of the breas t areola as a secondary mound above the contour of the breast and pubic hair of adult texture and color with no spread to the medial surface of Pediatrics 01May2009DO NOT DISTRIBUTE - 49 -the thighs. This patient s examinati on is most consistent with which Tanner stage of development?a) Stage I b) Stage IIc) Stage IIId) Stage IVe) Stage V75) Which of the following statements about acute myeloid leu emia (AML) is true?a) The preferred treatment for all types of AML is bone marrow transplant b) Chemotherapy used for AML is more intense than that used for ALLc) Hyperleu ocytosis is not a problem with AMLd) Patients with Down syndrome and AML have a worse prognosise) Secondary AML has a good respons e to therapy76) A 13-year-old male patient presents with intermittent abdominal pain, diarrhea,weight loss, and growth failure, and is noted on colonoscopy to h ave inflammatory s iplesions throughout the colon with rectal sparing. Which of the following is true?a) Ulcerative colitis typically is characterized by rectal sparing b) Ulcerative colitis typically is characterized by s ip lesionsc) Croh n disease typically is characterized by transmural diseased) Ulcerative colitis typically is associated with growth failuree) Ulcerative colitis typically is as sociated with perianal disease77) You are moonlighting in the pediatric emergenc y department when a 10-year-oldmale arrives by ambulance with lethargy, confusio n, dizziness, and a severe headache.His parents and maternal grandmother are in the adult emergency department with lesssevere but similar symptoms. The emergen cy medical technicians report that they werecalled by the police who found the f amily sleeping in their car with the engine running attheir Christmas tree stand . Carbon monoxide poisoning is suspected. Which of thefollowing should be the fi rst step in the evaluation and management of this patient?a) Obtain an EKG b) Dr aw an arterial blood gasc) Draw a blood carboxyhemoglobin leveld) Administer 100 % oxygene) Stabilize the patient for transfer to a hyperbaric oxygen chamber 78) A 5-year-old girl presents to the emergency department with a 12-hour history o f fever and respiratory distress. On physical examination, the girl appears toxi c, isdrooling, and leaning forward with her chin extended. She has a temperature of 104F(40C), and a respiratory rate of 32 breaths/minute. Which of the following is the mostli ely diagnosis?a) Epiglottitis b) Croupc) Bacterial pneumoniad) Di phtheriae) Anaphylaxis79) In response to your question concerning guns in the ho me during a routine adolescenthealth maintenance visit, the mother of the patien t tells you that her husband, the boy s Pediatrics 01May2009DO NOT DISTRIBUTE - 50 -stepfather, eeps a loaded handgun i n the bed table drawer for protection. You would becorrect in telling this famil y that an adolescent who lives in a home with a gun:a) Is less li ely than peers who do not live with guns to die from homicide b) Is less li ely than peers who do not live with guns to commit suicidec) Is mature enough to use good safety p recautions, so storing the handgunseparately from the ammunition is unnecessaryd ) Has a 10-fold greater ris of dying from suicide than peers who do not live in homes with gunse) Is less li ely than peers who do not live with guns to be shot during a domesticdispute80) You are seeing a new patient for a health-maintenan ce visit. The child is able to tellyou his age and gender and spea s in five to eight word sentences. His grandmother tellsyou that he is able to pedal a tricyc le. He can perform a broad jump when the behavior ismodeled and is able to copy a circle. However, he cannot yet balance on one foot or copya cross. You record that the patient s developmental achievement is consistent with hisage. Which of the following most closely correlates with this child s age in years?a) 2-years b) 3-yearsc) 4-yearsd) 5-yearse) 6-years81) A child weighing 27 g with a histo ry of vomiting for 36 hours is judged to be 10%dehydrated based on vital signs a nd physical examination. The serum sodiummeasurement is 134 mEq/L. An initial 54 0-mL bolus of normal saline results instabilization of the heart rate and improv ed capillary refill. Which of the following is themost appropriate parenteral fl uid choice for the next 8 hours?a) D5 0.2 normal saline with 20mEq/L KCl (added

after urination) at 120mL/hr b) D5 0.2 normal saline with 20mEq/L KCl (added af ter urination) at 180mL/hr c) D5 0.2 normal saline with 20mEq/L KCl (added after urination) at 220mL/hr d) D 0.45 normal saline with 20mEq/L KCl (added after ur ination) at 220mL/hr e) D5 0.45 normal saline with 20mEq/L KCl (added after urin ation) at 180mL/hr 82) An 8-year-old boy presents with growth failure and vague abdominal pain. Theabdomen is distended. There is no perianal disease, abdominal mass, or tenderness. Thenext set of diagnostic tests should include:a) CBC, CRP , tissue transglutaminase assay b) CT scan of the abdomenc) Urinalysis, sweat ch loride, laparotomyd) Colonoscopy, upper endoscopye) Stool culture for ova and pa rasites83) A 3-year-old boy presents to the pediatrician with fever, pallor, ano rexia, joint pain, petechiae, and hepatosplenomegaly. Which of the following is the most li ely diagnosis?a) Acute lymphoblastic leu emia b) Acute myelogenous l eu emiac) Juvenile chronic myelogenous leu emiad) Aplastic anemiae) Osteosarcoma Pediatrics 01May2009DO NOT DISTRIBUTE - 51 -84) A 16-year-old girl who is 2 year s post-menarche presents with mildly unevenshoulders and a small degree of one-s ided rib prominence. Radiographs reveal a 25-degree scoliosis. Which of the foll owing represents the best treatment?a) Posterior spinal fusion b) Intensive phys ical therapyc) Scoliosis bracingd) Spinal manipulatione) Observation with repeat x-ray in 1 year 85) A 5-year-old boy who returned from a camping trip to his gr andparents farm inVirginia develops a fever of 103F, a headache, vomiting, and a n erythematous, macular rash on his wrists and an les. On physical examination, he is moderately tachycardic withotherwise stable vital signs and no focal signs of infection. A CBC reveals a normalWBC count and differential and normal hemog lobin. However, the boy s platelet count is65,000/mm3. Serum electrolytes are no rmal. Blood cultures and immunofluorescentstudies are sent. Which of the followi ng is the most appropriate next course of action?a) Discharge home on amoxicilli n with close follow-up and reliable caregivers b) Discharge home on amoxicillinclavulanic acid with close follow-up andreliable caregiversc) Hospitalization fo r observation pending further test resultsd) Hospitalization for intravenous dox ycycline and cefotaximee) Hospitalization for intravenous doxycycline86) A 5-yea r-old boy presents with painful swelling of the hand and feet since the day befo re. Since earlier today, he has palpable purpura on the lower extremities, and a lsodeveloped intermittent, colic y midabdominal pain. Prior to these events, he had a coldfor 1 wee . He did not have fevers, and overall is well appearing. On physicalexamination, he has normal vital signs. He has palpable purpura on the l ower extremitiesand buttoc s. He has scrotal swelling. His hand and feet are puf fy, and he has pain withmovement of the an le joints. His abdominal examination is unremar able. A complete blood count shows normal results with a platelet cou nt of 350,000/mm3. Which of thefollowing laboratory tests is most often abnormal in this disease process?a) Antinuclear antibody (ANA) b) Antineutrophil cytopla smic antibody (ANCA)c) Complement C3 and C4 levelsd) Urinalysise) Serum creatini ne87) A 12-month-old male infant presents with a hemoglobin of 7.5 and a hematoc rit of 22%. The mean corpuscular volume is 65 and the adjusted reticulocyte coun t is 1.0%.What is the most li ely cause of anemia in this child?a) Iron-deficien cy anemia b) Anemia of chronic diseasec) Transient erythrocytopenia of childhood d) Thalassemia syndromee) Parvovirus B19 aplastic crisis88) A 12-year-old male a dolescent presents with a 1-month history of fever, weight loss,fatigue, and pai n and localized swelling of the mid-proximal femur. Which of thefollowing is the most li ely diagnosis? Pediatrics 01May2009DO NOT DISTRIBUTE - 52 -a) Ewing sarcoma b) Osteosarcomac) C hronic osteomyelitisd) Benign bone tumor e) Eosinophilic granuloma89) You are ex amining a 3-year-old girl at her well-child visit. While she is staring at her s tuffed cow in your hands, you quic ly cover her right eye with an index card. Wh en theindex card is removed seconds later, you notice that the right eye drifts ba c toward thecenter. This reaction in response to the cover test indicates what abnormal condition?a) Strabismus b) Amblyopiac) Leu ocoriad) Retinoblastomae) Na solacrimal duct obstruction90) A 14-year-old girl is brought to your office by h er mother because she is complainingof seeing double. The history is significant f or headaches that wa en the patient fromsleep in the morning but are relieved by

vomiting. On physical examination, you note thatshe is unable to abduct either eye. Lower extremity reflexes are slightly exaggerated.Which of the following ph ysical signs is most li ely to be present in this patient?a) Hypotension b) Papi lledemac) Tachycardiad) Patency of the anterior fontanellee) Erythema migrans91) A previously healthy 4-year-old girl presents with a history of diarrhea and vo mitingfor the past 3 days and decreased urine output for the past 12 hours. On e xamination, shehas a heart rate of 120 beats per minute, blood pressure of 105/6 5 mm Hg, and no edema.The blood tests reveal serum sodium of 128 mEq/L, potassiu m 5.6 mEq/L, bicarbonate12 mEq/L, BUN 55 mg/dL, creatinine 1.6 mg/dL. The urine tests reveal a fractionalexcretion of sodium of 0.1. The idney ultrasound is no rmal. Which of the followingconstitutes the most appropriate immediate managemen t of this child s acute renalfailure?a) Intravenous normal saline bolus to corre ct the renal hypoperfusion b) Intravenous bicarbonate to correct the metabolic a cidosisc) Intravenous furosemide to correct the fluid overloadd) Intravenous ant ibiotics to correct the infectious gastroenteritise) Initiation of dialysis to c orrect the acute renal failure92) A very tired mother brings her 6-wee -old infa nt to your office because he screamsfor hours and hours a day and nothing ma es h im stop. His parent describes the cryingspells as occurring daily and lasting sev eral hours, usually through the late afternoon andearly evening. Nothing seems t o console the child during these episodes. While he iscrying, the infant often p ulls his nees to his abdomen as if he is in pain. Other than thecrying spells, the child is asymptomatic. He feeds well and moves his bowels regularly.The chil d s weight, length, and head circumference are normal, and his physicalexaminati on is normal. This patient s history and physical examination are mostconsistent with which of the following conditions? Pediatrics 01May2009DO NOT DISTRIBUTE - 53 -a) Feeding intolerance b) Cow mil p rotein allergyc) Intussusceptiond) Hirschsprung diseasee) Colic93) A newborn inf ant has a slight hip clic on hip examination. Which of the followingris factor s would most strongly support further evaluation?a) Female patient b) First born c) Torticollisd) Metatarsus adductuse) Breech presentation or family history of developmental dysplasia of the hip94) A 14-year-old patient in your practice wit h anorexia nervosa has fallen to 80% of her ideal body weight for height and gen der. She has not menstruated in 9 months. She has postural hypotension and a low heart rate. Which of the following murmurs is most li elyto be present on this patient s cardiac examination?a) A midsystolic clic , followed by a murmur b) A fixed split S2c) A vibratory holosystolic murmur in both axillad) A third heart sounde) A nonspecific ejection murmur at the base of the heart95) You are offer ing preventive counseling to the parent of a 12-month-old child at ahealth maint enance visit. The child weighs 18 lbs. You would be correct in informing the par ent that this child should be:a) Restrained in a rear-facing infant car seat in the bac seat of the car until he hasreached 20 lbs. in weight b) Restrained in a forward-facing infant car seat in the bac seat of the car sincehe is now#1 ye ar of agec) Restrained in a rear-facing infant car seat in the front seat of the car until he hasreached 20 lbs. in weightd) Restrained in a forward-facing infa nt car seat in the front seat of the car sincehe is now#1 year of agee) Restrain ed in a forward-facing booster seat in the bac seat of the car since heis now#1 year of age96) A 4-wee -old male infant born at term presents with emesis, dehy dration, and poor weight gain. The pediatrician evaluating the child palpates an olive-sized mass in thechild s epigastrium. She believes the infant may have py loric stenosis. Which of thefollowing clinical presentations is most consistent with pyloric stenosis?a) Projectile nonbilious emesis b) Bilious emesisc) Bloody diarrhead) Violent episodes of intermittent colic y pain and emesise) Right low er quadrant abdominal pain97) A 5-year-old boy presents to the emergency departm ent with complaints of dizzinessand confusion. Three days before presentation he developed a low-grade fever andvomited twice. Since then, the fever and vomitin g have resolved, but the patient has Pediatrics 01May2009DO NOT DISTRIBUTE - 54 - passed 8 to 10 loose, foul-smelling stools per day. The boy s mother has been afraid togive him anything but water or diluted juice due to his history of vomiting. Deep tendonreflexes are diminis hed throughout. This patient s ataxia and confusion are most li elydue to which

of the following electrolyte imbalances?a) Hypomagnesemia b) Hyper alemiac) Meta bolic al alosisd) Hypochloremiae) Hyponatremia98) A 13-year-old male presents to the office with short stature. Growth datademonstrates that he has been growing between the 3rd and 5th percentile at a steady ratesince age 4 years. His fathe r started shaving at age 17 and completed his growth at age 19years. What examin ation and wor up would support the diagnosis of constitutional delayof growth an d puberty?a) Acne and axillary hair, Tanner III pubic hair, testicular volume 12 cc, bone age14 years, TSH 1.5 (0.5 to 4.8), IGF-I 340 (152 to 540) b) No axilla ry hair, Tanner I pubic hair, testicular volume 4 cc, bone age 11 years,TSH 12 ( 0.5 to 4.8), IGF-I 200 (152 to 540)c) Scant axillary hair, Tanner II pubic hair, testicular volume 5 cc, bone age 11years, TSH 2.1 (0.5 to 4.8), IGF-I 420 (152 to 540)d) No axillary hair, Tanner I pubic hair, testicular volume 4 cc, bone ag e 11 years,TSH 3.1 (0.5 to 4.8), IGF-I 62 (152 to 540)99) A 24-month-old male in your office for his regular health maintenance visit has thefollowing results o n screening tests: hemoglobin 9.6 g/dL; capillary blood lead level 16mcg/dL. He lives in Section 8 housing in poor repair built before 1960. Which of thefollowi ng is the most appropriate next course of action?a) Counsel the family regarding lead removal and rechec the level in 6 months b) Refer the family to the local governmental lead management agencyc) Obtain a venous lead level for confirmati ond) Start the patient on oral succimer on an outpatient basise) Obtain neurodev elopmental testing for the patient100) A 10-year-old girl presents with a linear strea s of thic ened and indurated s in onthe right arm and trun . The linear s trea on the right arm has a longitudinal orientationand extends from the upper arm to the dorsal aspect of the hand, whereas the linear strea on the trun is transversely oriented. The lesions are surrounded by a halo of erythemawith a vi olaceous appearance. The central portion is hyperpigmented and thic ened.Which o f the following complications is this child most li ely to develop?a) Esophageal dysfunction b) Pulmonary fibrosisc) Contracture of the right elbowd) Raynaud ph enomenone) Digital necrosisJames Lamberg Pediatrics 01May2009DO NOT DISTRIBUTE - 55 -AnswerKeyPeds #11) A2) C3) E4) B5) D 6) A7) E8) B9) D10) C11) B12) A13) E14) C15) C16) A17) A18) E19) A20) B21) D22) F23) D24) D25) B26) B27) E28) B29) A30) A31) C32) D33) D34) A35) D36) C37) C38) D39) B40) B41) D42) E43) A44) F45) C46) E47) D48) C49) F50) A51) B52) C53) E54) E55) D56) A57) E58) B59) A60) D61) C62) D63) C64) D65) C66) APeds #21) C2) D3) C 4) B5) B6) A7) C8) B9) E10) D11) C12) B13) E14) B15) E16) D17) A18) C19) B20) A2 1) C22) D23) E24) B25) A26) C27) D28) F29) C30) C31) B32) A33) D34) E35) D36) C3 7) C38) C39) D40) E41) D42) B43) E44) D45) D46) D47) B48) D49) A50) C51) C52) A5 3) E54) C55) E56) D57) A58) A59) E60) C61) D62) A63) B64) E65) D66) APeds #31) C 2) D3) A4) A5) C6) C7) E8) D9) D10) B11) C12) C13) B14) A15) B16) D17) A18) D19) D20) C21) D22) E23) A24) D25) A26) B27) E28) A29) C30) A31) D32) C33) B34) D35) B36) A37) E38) C39) A40) A41) B42) B43) C44) D45) A Pediatrics 01May2009DO NOT DISTRIBUTE - 56 -46) D47) E48) D49) C50) B51) C52) D5 3) A54) E55) C56) E57) D58) A59) C60) C61) A62) E63) C64) E65) C66) D67) E68) DP eds #41) D2) D3) B4) C5) E6) A7) A8) B9) C10) E11) A12) D13) C14) A15) B16) B17) D18) C19) DPeds #51) B2) A3) D4) D5) A6) B7) A8) C9) A10) A11) C12) D13) B14) B 15) B16) E17) C18) A19) A20) B21) C22) D23) A24) B25) B26) D27) A28) C29) C30) E 31) D32) A33) A34) D35) B36) E37) C38) C39) D40) D41) E42) D43) E44) C45) C46) C 47) A48) C49) C50) D51) A52) C53) A54) D55) D56) B57) C58) B59) C60) B61) C62) B 63) B64) E65) C66) A67) D68) E69) B70) C71) A72) C73) B74) D75) B76) C77) D78) A 79) D80) B81) C82) A83) A84) E85) D86) D87) A88) A89) A90) B91) A92) E93) E94) A 95) A96) A97) E98) C99) C100) C of 56 Leave a Comment Comment must not be empty. You must be logged in to leave a comment.SubmitCharacters: 400 drmas

I FIND THIS DOCUMENT VERY USEFUL. HOW DO I GET A COPY? reply01 / 04 / 2010 MedShare Clic the "Download" lin on the top left to get the PDF 01 / 04 / 2010 calotz_chubie6891 hi... can you send this to me? at calotz_chubie@yahoo.com.. thnx! reply10 / 09 / 2009 MedShare You can download the PDF directly from here 10 / 24 / 2009 fatima fatima thx alot:) reply09 / 26 / 2009 Comment must not be empty. You must be logged in to leave a comment.SubmitCharacters: ...Quiz Pediatrics Quiz Pediatrics Download or Print 12,146 ReadsInfo and Rating Category: School Wor > Study Guides, Notes, & Quizzes Rating: (3 Ratings) Upload Date: 04/15/2009 Copyright: Attribution Non-commercial Tags: medical school medicine mcq mcqs quiz exammedical school medicine mcq mcqs quiz exam(fewer) Flag document for inapproriate content This is a private document. Uploaded by MedShare Follow DownloadEmbed DocCopy Lin Add To CollectionCommentsReadcastShare Tweet Share on Scribd: Readcast SearchTIP Press Ctrl-F F to quic ly search anywhere in the document. SearchSearch History: Searching... Result 00 of 0000 results for result for p. More from This UserRelated Documents More From This User 49 p.Study Notes Family Medicine Study Notes Family Medicine 86 p.Study Notes Pediatrics Study Notes Pediatrics 55 p.Study Notes Obstetrics Gynecology Study Notes Obstetrics Gynecology Next 101 p.Study Notes Internal Medicine Study Notes Internal Medicine 12 p.Study Notes Emergency Medicine Study Notes Emergency Medicine

58 p.Study Notes Anesthesiology Study Notes Anesthesiology Prev Next 1 p.Anesthesia Medications Anesthesia Medications 2 p.Anesthesia Care Plan Anesthesia Care Plan 50 p.Study Notes Psychiatry Study Notes Psychiatry Prev Next 56 p.Study Notes Surgery Study Notes Surgery 16 p.Study Hints Clinical Rotations Study Hints Clinical Rotations 34 p.Quic Medical Poc et Reference Quic Medical Poc et Reference Prev Next 91 p.Quiz Behavioral Science Quiz Behavioral Science 14 p.Quiz Spirituality Ethics Quiz Spirituality Ethics 123 p.Quiz Respiratory Quiz Respiratory Prev Next 78 p.Quiz Reproductive Quiz Reproductive 81 p.Quiz Renal Quiz Renal 26 p.Quiz Public Health Quiz Public Health Prev Next 46 p.Quiz Physiology Quiz Physiology 54 p.Quiz Pharmacology Quiz Pharmacology Part 42 p.Quiz Pharmacology Quiz Pharmacology Part

Prev Next 56 p.Quiz Pediatrics Quiz Pediatrics 28 p.Quiz Pathology Quiz Pathology 28 p.Quiz Osteopathic Part 4 of 4 Quiz Osteopathic Part 4 of 4 Prev Next 54 p.Quiz Osteopathic Part 3 of 4 Quiz Osteopathic Part 3 of 4 Prev Related Docuements 337 p.a Pediatrics From Hassan Jafry337 p.LANGE Q & a Pediatrics, 7th Edition-0071475680 From ala 5510 p.Case Files Pediatrics - Toy [2009]

Part 2 of Part 1 of

2 of 2 2 1 of 2 2

From summer_zephyr Next 510 p.Case Files - Pediatrics From darsews510 p.Case Files Pediatrics From Dra Tr 510 p.Case Files Pediatrics From Abdallah Darras Prev Next 510 p.Case Files - Pediatrics, 3rd Edition (LANGE Case Files) From sagoor138 p.MCCQE 2004 From adamfrever20 p.Self-Assessment Questions, Group 6 pg mcqs From naveen oval Prev Next 95 p.mccee7 From vugar7497 p.Pediatric Nursing Pediatric nursing notes f you have questions please email me> freenursingnot... From FreeNursingNotes15 p.Practice Questions From Tetet Valencia Prev Next 73 p.2011Step3 contents From alyssaucmd73 p.2010Step3 From s123432113 p.Pacema ers From mmdula Prev Next 11 p.iLearn From sturgeond997 p.1 Aparitia Asigurarilor Pe Plan National Si International From Sadic a Ni 1 p.LID:EZM - RNAV (GPS) RWY 02 (1009) RNAV (GPS) RWY 02 approach procedure for HEART OF GEORGIA RGNL From Terminal Procedures Prev Next 20 p.Acute Myeloid Leu emia From hemendre80 p.Overview From Pablo Osuna 80 p.Creating From Pablo Osuna Prev Next 80 p.MVC Music Store - Tutorial From Pablo Osuna 97 p.mary-boo From Red1 p.Senator Nina Turner From ohiosenatedemocrats Prev

the StoreManagerViewModel - V0.8 how tp - ENewsletter September 2010

Anda mungkin juga menyukai